Moving on from Cognito Mentoring

Back in December 2013, Jonah Sinick and I launched Cognito Mentoring, an advising service for intellectually curious students. Our goal was to improve the quality of learning, productivity, and life choices of the student population at large, and we chose to focus on intellectually curious students because of their greater potential as well as our greater ability to relate with that population. We began by offering free personalized advising. Jonah announced the launch in a LessWrong post, hoping to attract the attention of LessWrong’s intellectually curious readership.

Since then, we feel we’ve done a fair amount, with a lot of help from LessWrong. We’ve published a few dozen blog posts and have an information wiki. Slightly under a hundred people contacted us asking us for advice (many from LessWrong), and we had substantive interactions with over 50 of them. As our reviews from students and parents suggest, we’ve made a good impression and have had a positive impact on many of the people we’ve advised. We’re proud of what we’ve accomplished and grateful for the support and constructive criticism we’ve received on LessWrong.

However, what we’ve learned in the last few months has led us to the conclusion that Cognito Mentoring is not ripe for being a full-time work opportunity for the two of us.

For the last few months, we’ve eschewed regular jobs and instead done contract work that provides us the flexibility to work on Cognito Mentoring, eating into our savings somewhat to cover the cost of living differences. This is a temporary arrangement and is not sustainable. We therefore intend to scale back our work on Cognito Mentoring to “maintenance mode” so that people can continue to benefit from the resources we’ve already collected, with minimal additional effort on our part, freeing us up to take regular jobs with more demanding time requirements.

We might revive Cognito Mentoring as a part-time or full-time endeavor in the future if there are significant changes to our beliefs about the traction, impact, and long-run financial viability of Cognito Mentoring. Part of the purpose of “maintenance mode” will be to leave open the possibility of such a revival if the idea does indeed have potential.

In this post, I discuss some of the factors that led us to change our view, the conditions under which we might revive Cognito Mentoring, and more details about how “maintenance mode” for Cognito Mentoring will look.

Reason #1: Downward update on social value

We do think that the work we’ve done on Cognito Mentoring so far has generated social value, and the continued presence of the website will add more value over time. However, our view has shifted in the direction of lower marginal social value from working on Cognito Mentoring full-time, relative to simply keeping the website live and doing occasional work to improve it. Specifically:

  • It’s quite possible that the lowest-hanging fruit with respect to the advisees who would be most receptive to our advice has already been plucked. We received the bulk of our advisees through LessWrong within the month after our initial posting. Other places where we’ve posted about our service have led to fewer advisees (more here).
  • Of our website content, only a small fraction of the content gets significant traction (see our list of popular pages), so honing and promoting our best content might be a better strategy for improving social value than trying to create a comprehensive resource. This can be done while in maintenance mode, and does not require full-time effort on our part.

What might lead us to change our minds: If we continue to be contacted by large numbers of potentially high-impact people, or we get evidence that the advising we’ve already done has had significantly greater impact than we think it did, we’ll update our social value upward.

    Reason #2: Downward update on long-run financial viability

    We have enough cash to go on for a few more months. But for Cognito Mentoring to be something that we work full time on, we need an eventual steady source of income from it. Around mid-March 2014, we came to the realization that charging advisees is not a viable revenue source, as Jonah described at the end of his post about how Cognito Mentoring can do the most good (see also this comment by Luke Muehlhauser and Jonah’s response to it below the comment). At that point, we decided to focus more on our informational content and on looking for philanthropic funding.

    Our effort at looking into philanthropic funding did give us a few leads, and some of them could plausibly result in us getting small grants. However, none of the leads we got pointed to potential steady long-term income sources. In other words, we don’t think philanthropic funding is a viable long-term revenue model for Cognito Mentoring.

    Our (anticipated) difficulty in getting philanthropic funding arises from two somewhat different reasons.

    1. What we’re doing is somewhat new and does not fit the standard mold of educational grants. Educational foundations tend to give grants for fairly specific activities, and what we’re doing does not seem to fit those.
    2. We haven’t demonstrated significant traction or impact yet (even though we’ve had a reasonable amount of per capita impact, the total number of people we’ve influenced so far is relatively small). This circles back to Reason #1: funders’ reluctance to fund us may in part stem from their belief that we won’t have much social value, given our lack of traction so far. Insofar as funders’ judgment carries some information value, this should also strengthen Reason #1.

    What might lead us to change our minds: If we are contacted by a funder who is willing to bankroll us for over a year and also offer a convincing reason for why he/she thinks bankrolling us is a good idea (so that we’re convinced that our funding can be sustained beyond a year) we’ll change our minds.

      Reason #3: Acquisition of knowledge and skills

      One of the reasons we’ve been able to have an impact through Cognito Mentoring so far is that both Jonah and I have knowledge of many diverse topics related to the questions that our advisees have posed to us. But our knowledge is still woefully inadequate in a number of areas. In particular, many advisees have asked us questions in the realms of technology, entrepreneurship, and the job environment, and while we have pointed them to resources on these, firsthand experience, or close secondhand experience, would help us more effectively guide advisees. We intend to take jobs related to computer technology (in fields such as programming or data science), and these jobs might be at startups or put us in close contact with startups. This will better position us to return to mentoring later if we choose to resume it part-time or full-time.

      Knowledge and skills we acquire working in the technology sector could also help us design better interfaces or websites that can more directly address the needs of our audience. So far, we’ve thought of ourselves as content-oriented people, so we’ve used standard off-the-shelf software such as WordPress (for our main website and blog) and MediaWiki (for our information wiki). Part of the reason is that we wanted to focus on content creation rather than interface design, but part of the reason we’ve stuck to these is that we didn’t think we could design interfaces. Once we’ve acquired more programming and design experience, we might be more open to the idea of designing interfaces and software that can meet particular needs of our target audience. We might design an interface that helps people study more effectively, make better life decisions, or share reviews of courses and colleges, in a manner similar to softwares or websites such as Anki or Beeminder or Goodreads. There might also be potential for a more effective online resource that teaches programming than those in existence (e.g. Codecademy). It’s not clear right now whether there exists a useful opportunity of this sort that we are particularly well-suited to, but with more coding experience, we’ll at least be able to implement an idea of this sort if we decide it has promise.

      Reason #4: Letting it brew in the background can give us a better idea of the potential

      If we continue to gradually add content to the wiki, and continue to get links and traffic to it from other sources, it’s likely that the traffic will grow slowly and steadily. The extent of organic growth will help us figure out how much promise Cognito Mentoring has. If our wiki gets to the point of steadily receiving thousands of pageviews a day, we will reconsider reviving Cognito Mentoring as a part-time or full-time endeavor. If, on the other hand, traffic remains at approximately the current level (about a hundred pageviews a day, once we exclude spikes arising from links from LessWrong and Marginal Revolution) then the idea is probably not worth revisiting, and we’ll leave it in maintenance mode.

      In addition, by maintaining contact with the people we’ve advised, we can get more insight into the sort of impact we’ve had, whether it is significant over the long term, and how it can be improved. This again can tell us whether our impact is sufficiently large as to make Cognito Mentoring worth reviving.

      What “maintenance mode” entails

      1. We’ll continue to have contact information available, but will scale back on personalized advising: People are welcome to contact us with questions and suggestions about content, but we will not generally offer detailed personalized responses or do research specific to individuals who contact us. We’ll attempt to point people to relevant content we’ve already written, or to other resources we’re already aware of that can address their concerns.
      2. The information wiki will remain live, and we will continue to make occasional improvements, but we won’t have a time schedule of when particular improvements have to be implemented by.
      3. Existing blog posts will remain, but we probably won’t be making many new blog posts. New blog posts will happen only if one of us has an idea that really seems worth sharing and for which the Cognito Mentoring blog is an ideal forum.
      4. We’ll continue our administrative roles in the communities of existing Cognito Mentoring advisees
      5. We’ll continue periodically reviewing the progress of people we’ve advised so far: This will help us get a better sense of how valuable our work has been, and can be useful should we choose to revive Cognito Mentoring.
      6. We’ll continue to correspond with advisees we have so far (time permitting), though we’ll give more priority to advisees who continue to maintain contact of their own accord and those whose activities seem to have higher impact potential.
      7. We’ll try to get our best content linked from other sources, such as about.com: Sources like about.com are targeted at the general population. We can try to get linked to from there as an additional resource for the more intellectually curious population that’s outside the core focus of about.com.
      8. We’ll link more extensively to other sources that people can use: For instance, we can more emphatically point to 80,000 Hours for people who are interested in career advising in relation to effective altruist pursuits. We can point to about.com and College Confidential for more general information about mainstream institutions. We already make a number of recommendations on our website, but as we stop working actively, it becomes all the more important that people who come to us are appropriately redirected to other sources that can help them.

      Conclusion and summary (TL;DR)

      We (qua Cognito Mentoring) are grateful to LessWrong for being welcoming of our posts, offering constructive criticism, and providing us with some advisees we’ve enjoyed working with. We think that the work we’ve done has value, but don’t think that there’s enough marginal value from full-time work on Cognito Mentoring. We think we can do more good for ourselves and the world by switching Cognito Mentoring to maintenance mode and freeing our time currently spent on Cognito Mentoring for other pursuits. The material that we have already produced will continue to remain in the public domain and we hope that people will benefit from it. We may revisit our “maintenance mode” decision if new evidence changes our view regarding traction, impact, and long-run financial viability.

      Human capital or signaling? No, it’s about doing the Right Thing and acquiring karma

      There’s a huge debate among economists of education on whether the positive relationship between educational attainment and income is due to human capital, signaling, or ability bias. But what do the students themselves believe? Bryan Caplan has argued that students’ actions (for instance, their not sitting in for free on classes and their rejoicing at class cancellation) suggest a belief in the signaling model of education. At the same time, he notes that students may not fully believe the signaling model, and that shifting in the direction of that belief might improve individual educational attainment.

      Still, something seems wrong about the view that most people believe in the signaling model of education. While their actions are consistent with that view, I don’t think they frame it quite that way. I don’t think they usually think of it as “education is useless, but I’ll go through it anyway because that allows me to signal to potential employers that I have the necessary intelligence and personality traits to succeed on the job.” Instead, I believe that people’s model of school education is linked to the idea of karma: they do what the System wants them to do, because that’s their duty and the Right Thing to do. Many of them also expect that if they do the Right Thing, and fulfill their duties well, then the System shall reward them with financial security and a rewarding life. Others may take a more fateful stance, saying that it’s not up to them to judge what the System has in store for them, but they still need to do the Right Thing.

      The case of the devout Christian

      Consider a reasonably devout Christian who goes to church regularly. For such a person, going to church, and living a life in accordance with (his understanding of) Christian ethics is part of what he’s supposed to do. God will take care of him as long as he does his job well. In the long run, God will reward good behavior and doing the Right Thing, but it’s not for him to question God’s actions.

      Such a person might look bemused if you asked him, “Are you a practicing Christian because you believe in the prudential value of Christian teachings (the “human capital” theory) or because you want to give God the impression that you are worthy of being rewarded (the “signaling” theory”)?” Why? Partly, because the person attributes omniscience, omnipotence, and omnibenevolence to God, so that the very idea of having a conceptual distinction between what’s right and how to impress God seems wrong. Yes, he does expect that God will take care of him and reward him for his goodness (the “signaling” theory). Yes, he also believes that the Christian teachings are prudent (the “human capital” theory). But to him, these are not separate theories but just parts of the general belief in doing right and letting God take care of the rest.

      Surely not all Christians are like this. Some might be extreme signalers: they may be deliberately trying to optimize for (what they believe to be) God’s favor and maximizing the probability of making the cut to Heaven. Others might believe truly in the prudence of God’s teachings and think that any rewards that flow are because the advice makes sense at the worldly level (in terms of the non-divine consequences of actions) rather than because God is impressed by the signals they’re sending him through those actions. There are also a number of devout Christians I personally know who, regardless of their views on the matter, would be happy to entertain, examine, and discuss such hypotheses without feeling bemused. Still, I suspect the majority of Christians don’t separate the issue, and many might even be offended at second-guessing God.

      Note: I selected Christianity and a male sex just for ease of description; similar ideas apply to other religions and the female sex. Also note that in theory, some religious sects emphasize free will and others emphasize determinism more, but it’s not clear to me how much effect this has on people’s mental models on the ground.

      The schoolhouse as church: why human capital and signaling sound ridiculous

      Just as many people believe in following God’s path and letting Him take care of the rewards, many people believe that by doing the Right Thing educationally (being a Good Student and jumping through the appropriate hoops through correctly applied sincere effort) they’re doing their bit for the System. These people might be bemused at the cynicism involved in separating out “human capital” and “signaling” theories of education.

      Again, not everybody is like this. Some people are extreme signalers: they openly claim that school builds no useful skills, but grades are necessary to impress future employers, mates, and society at large. Some are human capital extremists: they openly claim that the main purpose is to acquire a strong foundation of knowledge, and they continue to do so even when the incentive from the perspective of grades is low. Some are consumption extremists: they believe in learning because it’s fun and intellectually stimulating. And some strategically combine these approaches. Yet, none of these categories describe most people.

      I’ve had students who worked considerably harder on courses than the bare minimum effort needed to get an A. This is despite the fact that they aren’t deeply interested in the subject, don’t believe it will be useful in later life, and aren’t likely to remember it for too long anyway. I think that the karma explanation fits best: people develop an image of themselves as Good Students who do their duty and fulfill their role in the system. They strive hard to fulfill that image, often going somewhat overboard beyond the bare minimum needed for signaling purposes, while still not trying to learn in ways that optimize for human capital acquisition. There are of course many other people who claim to aspire to the label of Good Student because it’s the Right Thing, and consider it a failing of virtue that they don’t currently qualify as Good Students. Of course, that’s what they say, and social desirability bias might play a role in individuals’ statements,  but the very fact that people consider such views socially desirable indicates the strong societal belief in being a Good Student and doing one’s academic duty.

      If you presented the signaling hypothesis to self-identified Good Students they’d probably be insulted. It’s like telling a devout Christian that he’s in it only to curry favor with God. At the same time, the human capital hypothesis might also seem ridiculous to them in light of their actual actions and experiences: they know they don’t remember or understand the material too well. Thinking of it as doing their bit for the System because it’s the Right Thing to do seems both noble and realistic.

      The impressive success of this approach

      At the individual level, this works! Regardless of the relative roles of human capital, signaling, and ability bias, people who go through higher levels of education and get better grades tend to earn better and get more high-status jobs than others. People who transform themselves from being bad students to good students often see rewards both academically and in later life in the form of better jobs. This could again be human capital, signaling, or ability bias. The ability bias explanation is plausible because it requires a lot of ability to turn from a bad student into a good student, about the same as it does to be a good student from the get-go or perhaps even more because transforming oneself is a difficult task.

      Can one do better?

      Doing what the System commands can be reasonably satisfying, and even rewarding. But for many people, and particularly for the people who do the most impressive things, it’s not necessarily the optimal path. This is because the System isn’t designed to maximize every individual’s success or life satisfaction, or even to optimize things for society as a whole. It’s based on a series of adjustments driven by squabbling between competing interests. It could be a lot worse, but a motivated person could do better.

      Also note that being a Good Student is fundamentally different from being a Good Worker. A worker, whether directly serving customers or reporting to a boss, is producing stuff that other people value. So, at least in principle, being a better worker translates to more gains for the customers. This means that a Good Worker is contributing to the System in a literal sense, and by doing a better job, directly adds more value. But this sort of reasoning doesn’t apply to Good Students, because the actions of students qua students aren’t producing direct value. Their value is largely their consumption value to the students themselves and their instrumental value to the students’ current and later life choices.

      Many of the qualities that define a Good Student are qualities that are desirable in other contexts as well. In particular, good study habits are valuable not just in school but in any form of research that relies on intellectual comprehension and synthesis (this may be an example of the human capital gains from education, except that I don’t think most students acquire good study habits). So, one thing to learn from the Good Student model is good study habits. General traits of conscientiousness, hardwork, and willingness to work beyond the bare minimum needed for signaling purposes are also valuable to learn and practice.

      But the Good Student model breaks down when it comes to acquiring perspective about how to prioritize between different subjects, and how to actually learn and do things of direct value. A common example is perfectionism. The Good Student may spend hours practicing calculus to get a perfect score in the test, far beyond what’s necessary to get an A in the class or an AP BC 5, and yet not acquire a conceptual understanding of calculus or learn calculus in a way that would stick. Such a student has acquired a lot of karma, but has failed from both the human capital perspective (in not acquiring durable human capital) and the signaling perspective (in spending more effort than is needed for the signal). In an ideal world, material would be taught in a way that one can score highly on tests if and only if it serves useful human capital or signaling functions, but this is often not the case.

      Thus, I believe it makes sense to critically examine the activities one is pursuing as a student, and ask: “does this serve a useful purpose for me?” The purpose could be human capital. signaling, pure consumption, or something else (such as networking). Consider the following four extreme answers a student may give to why a particular high school or college matters:

      • Pure signaling: A follow-up might be: “how much effort would I need to put in to get a good return on investment as far as the signaling benefits go?” And then one has to stop at that level, rather than overshoot or undershoot.
      • Pure human capital: A follow-up might be: “how do I learn to maximize the long-term human capital acquired and retained?” In this world, test performance matters only as feedback rather than as the ultimate goal of one’s actions. Rather than trying to practice for hours on end to get a perfect score on a test, more effort will go into learning in ways that increase the probability of long-term retention in ways that are likely to prove useful later on. (As mentioned above, in an ideal world, these goals would converge).
      • Pure consumption: A follow-up might be: “how much effort should I put in in order to get the maximum enjoyment and stimulation (or other forms of consumptive experience), without feeling stressed or burdened by the material?”
      • Pure networking: A follow-up might be: “how do I optimize my course experience to maximize the extent to which I’m able to network with fellow students and instructors?”

      One might also believe that some combination of these explanations applies. For instance, a mixed human capital-cum-signaling explanation might recommend that one study all topics well enough to get an A, and then concentrate on acquiring a durable understanding of the few subtopics that one believes are needed for long-term knowledge and skills. For instance, a mastery of fractions matters a lot more than a mastery of quadratic equations, so a student preparing for a middle school or high school algebra course might choose to learn both at a basic level but get a really deep understanding of fractions. Similarly, in calculus, having a clear idea of what a function and derivative means matters a lot more than knowing how to differentiate trigonometric functions, so a student may superficially understand all aspects (to get the signaling benefits of a good grade) but dig deep into the concept of functions and the conceptual definition of derivatives (to acquire useful human capital). By thinking clearly about this, one may realize that perfecting one’s ability to differentiate complicated trigonometric function expressions or integrate complicated rational functions may not be valuable from either a human capital perspective or a signaling perspective for students not intending to go into a mathematically intensive discipline.

      Ultimately, the changes wrought by consciously thinking about these issues are not too dramatic. Even though the System is suboptimal, it’s locally optimal in small ways and one is constrained in one’s actions in any case. But the changes can nevertheless add up to lead one to be more strategic and less stressed, do better on all fronts (human capital, signaling, and consumption), and discover opportunities one might otherwise have missed.

      Economics majors and earnings: further exploration

      In Earnings of economics majors: general considerations I presented data showing that economics majors make substantially more money (20%-50%+) than majors in other liberal arts. I gave five hypotheses, each of which could partially account for the wage gap. These are possible differences between the majors in:

      1. Human capital acquisition.
      2. Acquisition of a desire to make money.
      3. Pre-existing ability as measured by tests.
      4. Pre-existing desire to make money.
      5. Signaling.

      I discussed a priori reasons for believing that they might be significant, and how one might go about testing the hypotheses and the extent to which they explain the wage gap.

      Having examined available data, I believe that with the possible exception of #3, based on publicly available information, there’s a huge amount of uncertainty as to the roles of these factors in explaining the wage gap. In many cases there is data suggesting the presence of effects, but the data is not robust and the sizes of the effects are entirely unclear. Furthermore, the hypotheses are not exhaustive: other factors (such as those mentioned at the very end of this post) plausibly play a role, making it difficult to reason in the fashion “factors A, B and C play very small roles, therefore factor D must play a large role.”

      I was originally hoping that there would be a simple, clearcut case for or against majoring in economics increasing earnings (relative to other liberal arts), but resolving the question would seem to be a major research project. Still, I hope that this post can help students who are contemplating majoring in economics or another liberal art get a feel for the “lay of the land,” and some of the points therein may be actionable for particular individuals.

      I’ll address each hypothesis in turn.

      This post is very long. If you’re short on time or attention, consider scanning over the subtopic headings and reading the sections that look most interesting. As usual, I’d appreciate any relevant thoughts, particularly if you’re a former economics major.

      #1 Human capital acquisition

      • I was unable to find systematic research on the role of human capital acquisition in economics majors’ greater earning power, and the available data is not robust.
      • There are major biases that make it difficult to infer the role of human capital acquisition in economics majors’ higher earning power from what people say.
      • It seems relatively uncommon for people to argue that object level knowledge of economics is useful on the job.
      • Many sources argue that economics majors indirectly learn skills such as analytical thinking and quantitative literacy that help on the job.
      • There may be considerable heterogeneity with respect to how useful particular courses for the economics major are, and how useful they are for a given career, across courses and careers. Estimating the magnitude of human capital acquisition from majoring in economics would require (i) subject matter knowledge of the individual courses (ii) knowledge of how common it is for an economics major to take each of these courses and (iii) how former economics majors are distributed across jobs.

      Biases affecting reports of the usefulness of economics on the job

      People’s opinions vary concerning how useful knowledge of economics is on the job. It’s difficult to assess the veracity of their views:

      • Representatives of economics departments are motivated to portray economics as more useful than it is.
      • When people make a decision, they often want to believe that they made the right decision, and will come up with rationalizations for why their decisions were right. Psychologist Robert Cialdini’s discusses this in his book Influence: The Psychology of Persuasion. Thus, former economics majors may be motivated to believe that what they learned in majoring in economics is useful on the job, and so exaggerate the usefulness of majoring in economics. In the other direction, those who didn’t major in economics may be motivated to believe that economics doesn’t teach employable skills.
      • People who major in economics may unknowingly pick up analytical / critical thinking skills that are superficially unrelated to their coursework, that prove useful on the job.
      • Those who major in economics often don’t have knowledge of what skills they would have picked up on had they majored in a different subject, and so may not be able to make a good comparison.
      • Some of those who express views on the subject may not know much about economics coursework. For example, employers who don’t know economics may notice that economics majors are better employees and misattribute this to what they learned in their economics classes, when it’s actually due to other factors.
      • It’s unclear how representative those who speak up on the subject are: people who have found economics useful on the job may be more or less likely to report on this than those who don’t.

      Quotations from current economics majors

      Some quotations from a message board thread on College Confidential. Note that at least two of the commenters hadn’t yet graduated from college at the time when they wrote.

      From VMadden

      Econ teaches you a lot of math which is very useful for careers in finance or accounting. The problem is that most of the later econ courses are highly theoretical and, therefore, not very appropriate to real life.

      From onhcetum

      It’s a bunch of theoretical ********. I am about four classes away from graduating, and I still haven’t learned crap. I know the basics of supply and demand and the relationship between the unemployment rate and inflation and GDP, but not a whole else.

      From econoboy

      Economics is an interesting field of study (like most social sciences) but it’s also (like most social sciences) not marketable and very academic in nature. Economics makes many simplifying assumptions about human behavior that in reality makes it very unrealistic. I’m getting ready to graduate college feeling like I have learned a lot and had a good well-rounded education but I’m also sitting here thinking to myself – “What am I going to do for a career?” I just put in 4 years of school and I honestly don’t feel much more marketable than a high school graduate.

      Quotations from former economics majors

      I had difficulty finding relevant quotations from former economics majors, who are the ones who would be best equipped to make an assessment.

      Found on the thread Why you should major in Economics at College Confidential:

      ”Pooja Jotwani, a recent graduate of Georgetown University in Washington D.C., […] says the major strengthened her business skills and provided her with something very simple: “financial security.””

      […]

      ”I work at the corporate headquarters of Mervyn’s as a financial analyst and internal auditor. None of my work relates to the material I studied as an undergrad, but that does not imply that my economics degree provides little value. As I have found from my own recruiting experience, an economics degree enjoys a great deal of respect because of exposure to issues related to the economy and thus business, as well as the analytical thinking that underlies economics.”

      […]

      ”I am now in Law School at Harvard. Nowadays, an Economics degree is one of the most helpful degrees to have for an entering law student. In my experience, it is the only academic non-legal subject that every substantive introductory law class teaches; every class thereafter assumes one has an understanding of economics.

      […]

      Well considering that I’m currently a Marketing intern for a video game company… and am working on marketing campaigns for a THQ game and several MMOs, I would say the Econ major on my resume didn’t hold me back from it. Also, the other marketing intern here is an Econ major from another school. Not going to say it helps for marketing, but it doesn’t hurt.”

      Quotations from college economics departments

      A few representative statements below. These sources are obviously biased.

      University of Houston

      Your economics training provides you with a terrific set of job skills, and in fact the economics major provides you with virtually all of the top ten most important job skills […] the graphs in economics represent quantitative concepts, and as an economics major you will certainly have no fear of graphs. Further, many classes use explicit numerical problem solving. You also have the opportunity to explicitly learn a wide range of statistical and computing tools […] There is no better major for learning analytical problem solving than economics. […] All of business is problem solving, and this is the expertise you have learned from the logical constructs in economics

      George Mason University

      When you graduate with an economics degree, you will have a good understanding of the national economy and will be able to think critically about problems in the business world. You will also have good communication skills and skills in manipulating data and using computer technology. Thus you will be prepared for many careers. […] Economics majors compete very well against most business majors for jobs in the business world. Many large corporations value the broad analytical training received by economics majors.

      Smith College

      Individuals with training in economics are well prepared for a variety of careers. In most cases, the rigor and precision of technical training in economics will give the graduate a competitive advantage.

      Quotations from economists

      As I mentioned in my original post, economist Bryan Caplan wrote:

      In my experience, undergraduate econ majors learn only two skills they’re likely to use in any job outside the Ivory Tower: (a) how to calculate a present discounted value, and (b) basic statistics. Except in top schools, I doubt most econ majors master either (a) or (b). The remainder of the economics curriculum simply isn’t vocational.

      Economist Greg Mankiw seems to think that majoring in economics increases earnings based on his blog post Why Major in Economics?. He doesn’t give a causal pathway, but could think that a causal pathway is human capital acquisition (or signaling).

      What employers say

      Surveys of employers are focused around how desirable they find economics majors, and most of the ones that I’ve found don’t separate out the extent to which economics majors are considered desirable because of what they learn in their economics major. But to the extent that employers want economics majors, it may be because of human capital acquisition, so employers wanting economics majors is evidence that employers think that economics majors acquire more human capital than usual, which is in turn (inconclusive) evidence that economics majors do in fact acquire more human capital than usual. For information on how desirable employers find economics majors, see the section of this post labelled “#5 Signaling”.

      The report Economics Graduates’ Skills and Employability has some information on how employers who are hiring students for roles that utilize economics specifically feel about how well prepared their starting employees are.

      The starting salary vs. midcareer salary test

      In my earlier post, I suggested that by comparing the percent by which starting salary grows over the course of a career for economics majors vs. other majors, one can obtain evidence for or against majoring in economics increasing human capital (relative to other majors). Specifically, I said that an increase in human capital should become less significant in relative terms over time (e.g. because the amount of human capital that one gains on the job over a course of a career should dominate what one gains in college), so that if majoring in economics increased human capital, one would expect the percentage growth for economics majors to be lower than those of other majors.

      Payscale’s 2008 report gives the percentage increase from starting salary to midcareer salary by major:


      Philosophy 103.5
      Economics 96.8
      PoliSci 91.7
      History 81.1
      English 70.3
      Psychology 68.2
      Sociology 59.5

      By the logic of my earlier post, the fact that the figure for economics is higher than that of most liberal arts would seem to be evidence that economics builds less human capital than the other majors do.

      There’s a confounding factor: economics majors have higher starting salaries, and in general, outside of technical professions such as engineering, people with higher starting salaries may experience higher percentage growth. (For an example supporting this intuition, my impression is that there are elite finance firms where earnings go up by 300% over the course of their careers, even after taking into account people being laid off.)

      To try to control for this factor, one could compare percentage growth by major with percentage growth figures for alumni of colleges with the same average starting salary as the major. The percentage growths for colleges with a fixed starting salary are highly variable, so employing this strategy would require detailed analysis. A cursory look at the data hints to me that the general effect of higher starting salary on percent increase of earnings is not high enough to fully account for economics majors having such a high percent increase relative to other majors. But since I haven’t done a systematic analysis, I have low confidence in this.

      So the fact that economics majors’ salaries increase by a much larger percentage than the other majors aside from philosophy would seem (very tentatively) to be evidence against the hypothesis that economics majors make more money because majoring in economics builds human capital.

      #2 Acquisition of a desire to make money

      Here too, there’s little data available.

      Increased selfishness?

      There are a number of psychology studies (see e.g. Does Studying Economics Breed Greed? and Are economists selfish? A lit review) that purport to associate studying economics with greed but:

      • In some cases there are explanations other than greed that could explain a difference between the populations. For example, one study finds that economists give less money to charity than other academics do, but this could reflect economists giving lower credence to charitable giving doing good (on account of being aware that it can distort the functionality of the market mechanism) rather than them being greedier.
      • Some of the findings are from artificial lab contexts that may be unrepresentative of the real world. For example, there are experiments in which economics students have defected more often than other students when playing prisoner’s dilemma, but Yezer, Goldfarb and Poppen have argued that this is unlikely to generalize to real world situations involving potential for cooperation.
      • As usual, there’s a potential issue of publication bias.
      • Publication bias and related issues are especially worrying in this context, because a lot of people have negative sentiments toward economists for a number of reasons: (i) people often disagree with economists’ political views as a class (ii) people are jealous of people who make more money than they do, and economics majors and economists make more money than others do (iii) those who are uncomfortable with quantitative subjects may be resentful that economists are facile with quantitative subjects (iv) economics involves harsh truths like the necessity of triage that many people find uncomfortable.

      Also, many of the studies don’t distinguish between testing “studying economics causes greed” and testing “those who are greedy are more likely to study economics” (though a few do).

      Subjective reports of the impact of majoring in economics on desire to make money

      One would like subjective reports of how studying economics altered students’ desire to make money. My original post attracted some relevant comments at The Bill Fold, which I’ve pasted below. They are few, and may be unrepresentative.

      moreadventurous:

      I don’t think majoring in Econ made me more selfish

      sherlock:

      I was an economics major, and I definitely have a desire to earn a high income. It’s an interesting question, whether or not the econ major influenced that, since I definitely didn’t feel that way coming into college. For me, I think that what changed was a much more acute awareness of what a raw deal everyone except the very highest earners are getting in this economy. Some of that came from studying economics in the midst of the great recession, sure, but I think the bigger factor was just experiencing the effects of income inequality firsthand in a way that I hadn’t before.

      DebtOrAlive

      I’m not convinced majoring in Econ increases selfishness so much as Econ majors (except those academia-bound) tend to be practical and focused on achieving at the very least a securely upper middle class lifestyle. [Looks around apt, looks outside at neighborhood, checks account balances] If it was supposed to make me more selfish, it didn’t exactly work, cause I’m not raking it in over here.

      diplostreetmix

      I would add that rather than increasing selfishness, a good Econ courseload should improve someone’s decision-making. You’ll wonder to yourself if you are being rational in your choices or maximizing your utility, which helps you decide what to do.

      dude

      Econ major here who chose a career in government, so to answer Mike’s question, no! Though I probably did use my Econ background to fully assess the relative merits of good salary + pension + benefits/number of hours per workweek. 😉

      Comparison of majors and nonmajors

      As I said in my original post, one would like a comparison between majors of how desire for money evolves over the course of college. Such a study may exist, but I haven’t been able to find it. The results of such a study would not be decisive: social desirability bias plausibly leads survey respondents to report that money matters less to them than it actually does, and a difference between economics majors and other majors could reflect different degrees of social desirability bias rather than different degrees of desire to make money.

      #3 Pre-existing ability as measured by tests

      Here we have relatively good data. Economics majors score higher on standardized tests than other liberal arts majors do, and this can be expected to partially account for the wage gap, but the standardized test gap by itself isn’t large enough to explain the wage gap.

      Average economic returns associated with higher SAT scores, and relative SAT scores of economics majors

      Table A3 of Estimating the Return to College Selectivity Over the Career Using Administrative Earning Data seems to indicate that a 100 point increase in SAT score (on a scale of 1600) was, for the class of 1976, associated with a ~2% increase in income, and was, for the class of 1989, associated with a ~4% increase in income.

      It’s been suggested that SAT math is what correlates with earnings rather than SAT verbal. If true, this would mean that a 50 point increase in math corresponds to the aforementioned percentages.

      Given that the percentage increased between the class of 1976 and the class of 1989, one would expect it to have increased further since then. If one extrapolates linearly, one gets a ~6% difference in income for the class of 2002.

      I found SAT score data from two sources, which I’ve given below. The data shows that economics majors score between 50 and 100 points higher than other majors, with more of the advantage in math than in verbal.


      Heterogeneity in human capital investments: High school curriculum, college major, and careers (Table 1) gives information on SAT score by major for students from 1993-2003. Some of the data therein is in the following table. The columns are major, math SAT, verbal SAT, combined SAT, and the difference between total SAT and economics total SAT.

      Economics 597 575 1172 0
      PoliSci 542 571 1113 -59
      History 558 595 1153 -19
      Psychology 530 540 1070 -102

      The 2009 paper Determining the Future Income of College Students gives figures as well, taken from a cohort of students who were between 13 and 17 in 1997. The columns are major, math SAT, verbal SAT, combined SAT, and the difference between total SAT and economics total SAT.


      Economics 629 588 1217 0
      PoliSci 573 580 1153 -64
      History 552 616 1168 -49
      English 562 609 1171 -46
      Psychology 554 579 1133 -84
      Sociology 486 508 994 -223

      Based on this data, there’s a lot of uncertainty as to how much of the wage gap comes from ability as measured by standardized tests. But the wage gap can’t be fully explained by differences in pre-existing ability as measured by standardized test scores. Consider the case of economics vs. political science, where economics majors make 20% more. If one assumes that a 50 point increase in math SAT increases income by 6%, and that economics majors have math SAT 75 points higher than political science majors, then one finds that pre-existing ability as measured by standardized tests would predict a 9% wage gap, which is substantially less than 20%.

      Comparison with graduates of elite colleges

      Another perspective that shows that ability as measured by standardized tests can’t be driving most of the wage gap comes from comparing the earnings of economics majors with the earnings of graduates of elite colleges.

      SAT scores of graduates of elite colleges are very high, with 75% of students scoring 700+ on a given section (at the 95th percentile or higher). The SAT scores of economics majors are lower: the highest subsection score listed above is 629 math, which is at the 85th percentile on the current SAT. This suggests that the 75th percentile of economics majors in SAT math is no more than the 93rd percentile for the general population (half way between 85th and 100th).

      Payscale reports that graduates of elite colleges make on the order of $50k-$60k and that economics majors make ~$48k starting salaries. According to the National Association of Colleges and Employers Salary Survey, the 25th/50th/75th percentiles of earnings of economics majors are $43k/$53k/$66k. There’s some incongruity between the results of the different surveys, but it seems likely that the 75th percentile economics majors in starting salaries make more than the median graduate of an elite university. This is further supported by Payscale’s 2008 report, which says that the 75th percentile of midcareer earnings for economics majors is ~$140k, compared with ~$120k/year for the median graduate of an elite college.

      So the 75th percentile economics majors have higher starting salary than median graduates elite colleges, despite having lower SAT scores. Furthermore, the median graduate of an elite college isn’t a randomly selected person with his or her SAT scores: he or she has other traits conducive to success (as measured by the college admissions process): despite these advantages, he or she makes less money than economics majors with lower SAT scores.

      Note: The juxtaposition of the midcareer data from Payscale’s 2008 report with present day SAT averages may give an impression that the gap is larger than it is, because at the time when people who are currently in the middle of their career were in college, SAT averages at elite colleges were lower than they are now.

      #4 Pre-existing desire to make money

      Economics majors probably have more desire to make money than other liberal arts majors do, but the size of the effect is unclear.

      Higher rates of selfishness?

      As discussed in the section “#2 Acquisition of a desire to make money,” there’s psychology research purporting to show that economics students and professors placing material gain above helping others, relative to other people. In some cases this research does not disentangle correlation and causation, leaving open the possibility that those who choose to study economics give more weight to having money. As above, there’s reason to be skeptical of this research.

      Evidence about the influence of financial motivations on major selection

      A number of research papers address how much weight college students give to future earnings when selecting a major. These papers vary in their results or in how they frame their results, or both.

      For example, How do young people choose college majors? by Montmarquette, Cannings, and Mahseredjian (2002) says

      The results of the paper show that the expected earnings variable is essential in the choice of a college major.

      whereas Ability Sorting and the Returns to College Major by Arcidiacono (2003) says

      Even after controlling for selection, large earnings premiums exist for certain majors. Differences in monetary returns explain little of the ability sorting across majors; virtually all ability sorting is because of preferences for particular majors in college and the workplace, with the former being larger than the latter.

      and Choosing the Field of Study in Post-Secondary Education: Do Expected Earnings Matter? by Beffy, Fougère and Maurel (2009) says

      Simulating for each given major a 10 percent increase in the expected earnings suggests that expected earnings have a statistically significant but quantitatively small impact on the allocation of students across majors.


      We’re interested in how much economics majors respond to financial incentives relative to other majors.

      The most relevant paper here seems to be Modeling college major choices using elicited measures of expectations and counterfactuals by Arcidiacono, Hotz, and Kang (2010). The sample size is small and possibly unrepresentative in relevant respects. The authors studied 173 undergraduates at Duke University, ~20% of whom were economics majors, and asked them questions involving them hypothetically majoring in other subjects. They grouped students into the categories of majors “science,” “humanities,” “engineering,” “social sciences,” “economics,” and “public policy.”

      They find that

      • Students in each major said that they would make more money if they were to major in economics than they would if they were to major in any other subject (Table 5).
      • When asked what they would major in if their expected earnings were the same across majors, only 16.6% said that they would major in economics, as opposed to the actual figure of 19.7% (Table 10).
      • Students of each major believed (on average) that they have the highest ability in their own major as opposed to others (Table 6).
      • When asked what they would major in if their ability levels were the same across subjects, 23.8% said that they would major in economics, as opposed to the actual figure of 19.7%.

      Taken together, the first two bullet points give the impression that ~15% of economics majors are in it for the money whereas 85% are not. If true, this would establish an upper bound on the desire of economics majors to make money relative to other majors.

      The third and fourth bullet points raise the possibility that some students in other majors are as responsive to financial incentives as the economics majors who are responsive to financial incentives, so that the actual gap in responsiveness to financial incentives could be smaller than the 15% figure suggests.

      As psychologist Robert Cialdini discusses in Influence: The Psychology of Persuasion, once people have made a decision, they tend to come up with multiple reasons why it was the right decision, unrelated to their original reason. It could be that of the 85% who said that they would major in economics anyway, for some, if they thought that their expected earnings were the same across majors before having decided on economics then they would have majored in something else. So the gap in responsiveness to financial incentives could be larger than the 15% figure suggests.


      A relevant study with a larger sample size is Choosing the Field of Study in Post-Secondary Education: Do Expected Earnings Matter? by Beffy, Fougère and Maurel. (I have not vetted their analysis, and don’t know whether or not it’s reliable.) They group majors into “Sciences,” “Humanities and Social Sciences” and “Law, Economics and Management.” They find that a 10% decrease in earnings (for a given subject group relative to others) reduces the percent of students who major in the subject group by 0.234 absolute percentage points for the sciences, 0.347 for the humanities and social sciences, and 0.432 for law, economics and management, suggesting that economics majors are more responsive to financial incentives. About 33% of students majored in each subject area, so the changes relative to number of majors in a given subject are on the order of 1%.

      One could interpret this as evidence that the students aren’t very responsive to financial incentives altogether, with law, economics and management students only marginally more than humanities and social science students. But it’s possible that one would see much a much greater diverge between the groups if one looked at the impact of a 20% or higher decrease in earnings rather than a 10% decrease in earnings. Perhaps what’s going on is that even if law, economics and management students made 10% less, they’d still be making more than they would in other majors, and they’d switch precisely at the point where they started making less money than they would in other majors.

      Since law, economics and management are grouped together, the gap could be coming from law and management rather than economics.

      #5 Signaling

      Majoring in economics probably signals desirable traits to employers, but the degree to which this is true, and the impact on expected earnings is unclear. (On the latter point, note that college major plausibly plays much less of a signaling role decisions about raises and promotions than it does in hiring decisions.)

      It’s worth noting that majoring in economics doesn’t have to be an accurate signal of a desirable trait in order for it to have signaling benefits: all that’s necessary is that employers believe that it’s a signal of the trait.

      The fact that economics majors make more is a signal that they’re better employees

      Unless one attributes the wage gap entirely to signaling (which would correspond to a belief that employers are generally mistaken in paying economics majors more), one can know that economics majors tend to be better workers, even if one doesn’t know why. Thus, employers will take the fact that economics majors make more money as a signal of quality. So the wage gap itself gives rise to signaling benefits.

      Employer surveys

      According to a survey of the The Chronicle of Higher Education:
      • Economics is one of the most desired majors in business (pg. 67)
      • 19% of employers state that they look for specific majors, 44% value some over others, 34% say that they balance it with other factors, and 3% say that it’s not important at all. (pg. 64).

      The preference for certain majors will be presumably be lower if one restricts to non-technical jobs. It should be noted that employers listed internships and employment during college as more significant than college major, with volunteer experience and extracurricular activities are not far behind. (pg. 24)

      2013 Gallup poll of business leaders asked about the weights that they assign to different factors when making hiring decisions. What they indicated is summarized below. (The columns starting with the second are the percent who ranked the factor as “very important”, “somewhat important” “not very important” and “not at all important”)

      Knowledge of field 84 14 2 0
      Applied skills in field 79 16 2 2
      College Major 28 42 22 8
      College attended 9 37 40 14

      Presumably it’s easier to assess knowledge of field and applied skills in field for experienced job candidates than for inexperienced job candidates, so that the role of college major in hiring decisions would decrease over time, as one would expect.

      Quotations from economics departments

      As with human capital acquisition, college economics department webpages generally say that employers consider economics majors to be desirable workers, and these sources are biased. They dominate Google hits when one searches for information on whether majoring in economics signals quality.

      Quotations from economists

      As above, economist Bryan Caplan thinks that economics teaches few employable skills, but also writes

      1. “Mickey Mouse majors” = Majors with low financial rewards. Judged against standard #1, economics is clearly not a Mickey Mouse major. Adjusting for preexisting ability, economics is one of the most lucrative majors – almost on par with electrical engineering.

      suggesting that he believes that being an economics major has significant signaling benefits.

      We asked economics students to identify majors as hard, moderate, or easy, and we found that 33 percent viewed economics as hard, 3 percent said sociology was hard, 7 percent saw psychology as hard, and 13 percent thought political science was hard. Since other social sciences were the primary alternative majors that most of the economics students considered, that data is compelling evidence that the respondents perceived those other majors as too easy. Students likely reasoned that taking a “too easy” major would signal to potential employers that the student had chosen an easy path through college, thereby hurting their chances of being hired.

      Quotations from former college students

      From Graduates Weigh In: What Is the Value of a Humanities Degree?

      Anton Troianovski:

      This attitude tracked with Harvard’s general message to freshmen, one that I think many of us accepted at face value: if you find the field of study that excites you most, the job search will take care of itself. Classmates would sometimes say they’d heard that Goldman Sachs and McKinsey may well hire a star of something like the folklore and mythology program over yet another cookie-cutter economics grad. This was in the booming mid-2000s. I wonder whether the harder economic times since then have made that message—emphasizing learning for learning’s sake—less convincing.

      Jessica E. Lessin:

      Most of the people I knew in college studied humanities. But they tried to make themselves more marketable to employers by squeezing in a class or two on economics. I think they were right to follow their interests. Recruiters at the time seemed more focused on how well you did in class, not what you studied. But if you wanted to work on Wall Street or at a consulting firm, an econ class didn’t hurt.

      From Economics, Once a Perplexing Subject, Is Enjoying a Bull Run at Universities

      For many students, however, economics is just the quickest way to land a good job. David Reinstein, 22 years old, started at George Washington University in a dual program of economics and political science. By the time he graduated in June, he had dropped the politics for economics alone. “People are impressed because it’s difficult,” Mr. Reinstein said. “Economics is respected.” His resume also impressed CNA Corp., an Arlington, Va., government contractor where Mr. Reinstein landed a job as a research specialist.

      Wall Street Oasis forum threads 

      These mostly concern getting jobs in finance:

      Liberal arts major= DEATH SENTENCE or NOT?,

      Do employers see a difference between an economics or a psychology major?

      Duke Econ Major?

      rejected twice !! Too liberal for the bank?!

      The starting salary vs. midcareer salary test

      The same considerations in the subsection “The starting salary vs. midcareer salary test” of the section “#1 Human capital acquisition” of this post apply to assessing the role of signaling.

      Other explanatory hypotheses

      • As diplostreetmix wrote at The Bill Fold, majoring in economics may not increase desire to make money relative to other desires, but rather, could help people learn to co-optimize to meet their goals better in general, and this could help them get higher paying jobs, even if the subject matter knowledge isn’t useful to them on the job.
      • There could be a correlation between finding economics interesting and finding certain jobs that happen to be high paying interesting, even in absence of a direct connection between the subject matter of economics and what’s done on the job, and independent of financial considerations. This would induce a correlation between majoring in economics and earning money.

      Earnings of economics majors: general considerations

      Some liberal arts majors make more money than others, but by far the ones who make the most are economics majors. The 2013-2014 Payscale Salary Report reports the following figures. The second column is median starting salary and the third is median mid-career salary, in thousands of dollars

      Economics 50 96
      Political Science 41 77
      Philosophy 39 78
      History 39 71
      English Literature 40 71
      Psychology 36 60
      Sociology 37 55

      This trend is robust, and I’ll give more supporting data as an appendix at the end of the post.

      The fact that economics majors make so much more is often taken to mean that majoring in economics raises future earnings. Is this true? In this post I’ll discuss some general considerations relevant to determining this, and discuss the sort of data that one might try use to resolve the question. In future posts, I’ll offer some such data, with analysis and discussion.

      I’d welcome any other ideas for testing the hypotheses, as well as pushback on the conceptual framework, and/or alternative hypotheses.

      As Bryan Caplan spells out in Economic Models of Education: A Typology for Future Reference, in general, a correlation between education and income can come from any of three things

      • Human capital acquisition: Education develops students’ employable skills.
      • Ability bias: Obtaining an educational credential reflects greater or lesser pre-existing ability (that exists independently of what’s learned in school), which is later reflected in earnings.
      • Signaling: An educational credential signals pre-existing ability (which as before, can be independent of what’s learned in school) which makes employers more likely to hire one.

      “Ability” here is best defined in a nonstandard way, as “traits conducive to making money.” An example of such a trait is intelligence, but there are other traits that are conducive to making money, such as the desire to succeed in lucrative careers, that don’t fit the standard definition of “ability.” Similarly, aside from acquiring human capital, one can also acquire other traits conducive to making money, such as the desire to make money. With this background in mind, consider the following hypotheses:

      1. Human capital acquisition — economics majors learn more employable skills in school than other the majors do, and this qualifies them for higher paying jobs.
      2. Acquisition of a desire to make money — Majoring in economics makes students want to make money more than they would have had they majored in other subjects, so that they seek higher paying jobs.
      3. Pre-existing ability as measured by tests  The people who choose to major in economics have higher standardized test scores than those who major in other subjects. This reflects underlying traits such as intelligence that make them perform better in job interviews, enable them to rise up the workplace ranks faster once hired and otherwise increase their earnings (for reasons not caused by their majoring in economics).
      4. Pre-existing desire to make money — The people who choose to major in economics are unusually determined to make money (or to succeed in lucrative careers for other reasons, which amounts to the same for our purposes). This will lead them to seek higher paying jobs (again, for reasons not caused by their majoring in economics).
      5. Signaling  Majoring in economics signals traits that employers find desirable.

      In principle, any combination of these hypotheses could explain why economics majors make more money. These are not necessarily exhaustive: there could be other factors that play roles, but we’ll restrict consideration to #1-#5 here.

      To the extent that #1, #2 and #5 explain the income gap, majoring in economics increases earnings (on average), and to the extent that #3 and #4 explain the income gap, majoring in economics does not increase earnings (on average). So for a high school or early college student who seeks to have high earnings, the question of the relative roles of these factors in explaining the earnings discrepancy is significant.

      While #1, #2 and #5 all point toward majoring in economics increasing earnings, they may have very different practical implications from one another. Some examples of this:

      • If the effect was all due to #1, then learning the material taught in the courses past the point needed for getting A’s could increase income, whereas if the effect was all due to #5, then learning the material past the point needed for getting A’s would carry no additional benefit.
      • If the effect was all due to #2, then for somebody who already gives top priority to making money, majoring in economics would not increase earnings.

      So determining the relative roles of #1, #2 and #5 can also be important.

      To assess the relative roles of #1-#5, we’ll discuss some a priori reasons why they might or might not explain the income gap, and what one would expect of real world data if a given factor played a major role.

      Human capital acquisition

      There’s a great deal of material online claiming that majoring in economics builds skills that employers value. For example, Economics: The Most Employable Liberal Art says

      While economics is technically classified as a liberal arts degree, it’s the only one with the distinction of offering both technical skills and a broad liberal arts education. That combination can prove to be very powerful, arming economics grads with in-demand knowledge in business, law, and math that look great on a resume, as well as soft skills that employers really want, including critical thinking, interpersonal abilities, and complex problem solving. Thanks to this powerhouse of technical-liberal skills, economics majors can find employment in a wide range of careers.

      “An economics degree opens a lot of doors,” says Dr. Anca M. Cotet with the Seton Hall Department of Economics and Legal Studies. Chris Mader, managing director of corporate accounts with Randstad agrees. “

      An education in economics is good preparation for almost any career path,” says Mader. “Economics majors generally have a holistic view of the business, and learn invaluable business skills and acumen that will be used throughout one’s career, no matter which industry they choose.” With these great opportunities, economics majors also enjoy a demand that puts them among the top majors by salary potential.

      However, one can also find characterizations of other majors that paint them in favorable terms from the point of view of building employable skills, and even if economics did compare favorably, that wouldn’t mean that economics builds employable skills to an appreciable degree: maybe no liberal arts degrees build employable skills to an appreciable degree.

      Economist Bryan Caplan has a different take from the people quoted above:

      In my experience, undergraduate econ majors learn only two skills they’re likely to use in any job outside the Ivory Tower: (a) how to calculate a present discounted value, and (b) basic statistics. Except in top schools, I doubt most econ majors master either (a) or (b). The remainder of the economics curriculum simply isn’t vocational.

      It could be that even if economics doesn’t teach material that’s directly relevant, studying economics indirectly builds skills such as critical thinking, which transfer over. Caplan comments on this, too:

      Many educators sooth their consciences by insisting that “I teach my students how to think, not what to think.” But this platitude goes against a hundred years of educational psychology. Education is very narrow; students learn the material you specifically teach them… if you’re lucky.

      To help resolve the question, ideally one would have a rich data set with anecdotal reports from former economics majors about how useful they’ve found what they learned in their coursework in their professional lives. Even this would not be decisive, because of people’s imperfect introspection.

      One possible test for whether majoring in economics builds employable skills comes from comparing the growth of economics majors’ salaries over the course of theirs career with the growth of other majors’ salaries over the course of their careers. Suppose that it were true that the skills that economics majors built by majoring in economics are as useful at the beginning of their careers as they are midcareer. Then if majoring in economics built employable skills, one would expect percent increase in wages to decrease over time. Why? Once people enter the work force, they begin accumulating more employable skills: possibly at a faster rate than they did while they were in college. Even if economics majors are 2 years ahead in training upon entering the workforce, while that gap would initially be highly significant, 10 years down the road it would be considerably less so.

      In practice, the skills that one learns in economics could become more useful (e.g. if higher level managerial type jobs use them more than entry level jobs) or less useful (e.g. because some people change fields of work over time, tending to move away from fields related to their major rather than toward fields related to their major). So there’s a lot of uncertainty about how a wage gap due to skills built from learning economics would evolve over the course of a career.

      But one should expect there to be a general drift tendency in the direction of what one learns in college mattering less over the course of one’s career (even if this drift tendency may be outweighed by other factors), so that if the wage gap decreases over time, that’s evidence in favor of majoring in economics building employable skills, and if it remains constant or increases over time, that’s evidence against majoring in economics building employable skills.

      Thus, by comparing percentage increase in wages across majors over the span of careers, one can obtain evidence for or against majoring in economics building employable skills.

      Acquisition of desire to make money

      Something that economics majors learn to a greater degree that those who major in other subjects is that income is a proxy to social value generated. Thus, majoring in economics will shift those who want to contribute social value from lower paying careers to higher paying careers.

      It’s also been suggested that studying economics increases people’s selfishness on account economic models assuming self-interested agents (see e.g.  Economics makes you selfish and Are economists selfish? A lit review). To the extent that this is true, those who would otherwise have chosen work based on the social value generated rather than income may give higher weight to income than to social value contributed when selecting a career. Here too, majoring in economics would shift people from lower paying careers to higher paying careers. The research on studying economics increasing selfishness has been criticized as nonrobust, and may not prove what it’s been purported to — I just raise it as one possibility of many.

      To investigate the possibility that majoring in economics increases desire to make money, one could survey recent college graduates about the evolution of their attitude toward making money over the course of college, and compare the results for economics majors with the results for liberal arts majors (perhaps also including questions on the role that they think their major played).

      Preexisting ability as measured by standardized tests

      SAT scores and other standardized test scores are known to correlate with earnings.

      One reason that this could be is that standardized test scores are a proxy to IQ, and IQ is known to correlate with earnings, with the correlation thought to be partially causal. Another reason could be that scores may be partially a measure of test preparation, which could correlate with conscientiousness, which is known to correlate with income, with the correlation thought to be largely causal. Standardized test performance also reflects the ability to remain focused for the duration of the test, which one would expect to correlate with job performance.

      Since the SAT is taken before college, SAT scores reflect ability that precedes majoring in economics. Thus, a difference in SAT scores between economics majors and the other majors would reflect a preexisting difference in ability between economics majors and the other majors. One can then compare the wage gap between economics majors and other majors with the wage gap between people who have SAT scores comparable to those of economics majors and SAT scores comparable to those of the other majors, to get a sense for the fraction of the wage gap that’s driven by the abilities measured by the SAT.

      The GRE and LSAT also pick up on the abilities measured by the SAT. These are taken toward the end of college, after a student has taken courses for his or her major and are therefore an a priori weaker measure of pre-existing ability, but data on how average scores on the GRE and LSAT vary by major still has some relevance to the degree to which the wage gap between economics majors and other majors is driven by pre-existing ability.

      The desire of economics majors to succeed in lucrative careers before selecting a major

      If students believe that majoring in economics will help them succeed in lucrative careers (irrespective of whether it actually does), then one will expect to find an unusually high concentration of people amongst economics majors who want to succeed in lucrative careers. These people will be more likely to take high paying jobs (relative to their peers in other majors, who may give more weight to factors such as work-life balance and enjoyment of work). This could partially explain why economics majors make more money later on.

      Why might students believe that majoring in economics will help them succeed in lucrative careers? Some possibilities:

      • Assuming that the correlation is causal — They may notice that economics majors make more money, and assume that the correlation between majoring in economics is causal.
      • Building human capital — They may think that economics builds employable skills more than other majors do. For example, they may mistakenly assume that the study of economics is the study of making money, because economics has something to do with money. Or, they may have object level knowledge that economics does in fact teach employable skills.
      • Signaling — See the section below.

      Furthermore, if students in economics want to make money more, then students who want to make money may want to major in economics more, as it gives them an opportunity to be around people like themselves (the major could serve as a better “cultural fit” for them).

      One could assess the relative desire of economics majors to make money by surveying students on why they chose their major, listing “financial returns” as one of the choices, and compare the strength and frequency with which economics majors give this weight, relative to other majors.

      The signaling value of majoring in economics

      If employers view economics majors more favorably than they view other majors, this will raise the expected earnings of economics majors. Some positive signals that majoring in economics might send to employers are:

      • Having built employable skills through studying economics. To the extent that employers believe (whether rightly or wrongly) that majoring in economics builds employable skills more than the other majors do, majoring in economics sends a positive signal.
      • Strong performance on what standardized tests measure. As above, this is correlated with earnings, and relatedly, job performance.
      • Desire to succeed in lucrative careers. Majoring in economics would signal this if economics majors are thought to have such a desire. As above, there are reasons to think that economics majors do disproportionately have this desire. Employers may believe (whether rightly or wrongly) that this is a desire will lead the job candidate to try to do a good job to get raises and promotions.
      • Work ethic. Majoring in economics would signal this trait if economics were thought to be a more difficult major than other liberal arts majors. My impression is that this is what people generally believe.

      One could assess the degree to which employers prefer economics majors by referring to survey data. Ideally, one would have survey data specifically on the strength of their preference for an economics degree over a degree in another liberal art, but in absence of this, one can look for data on (a) how much they say that a major matters and (b) whether they list economics as a preferred major.

      Analogous to the situation with human capital acquisition, there’s reason to think that the signaling benefits of majoring in economics decrease over time:

      • As one progresses through one’s career, one develops a history of work experience, and employable skills, and the role of these things relative to the role of what one majored in increases over time.
      • Once one has been hired, one’s major doesn’t figure in to decisions about promotions and raises.

      Based on this consideration, if the percentage by which economics majors are ahead decreases over the course of their careers, this is evidence in favor of signaling benefits playing a role in explaining the wage gap, whereas if the percentage remains the same or increases, this is evidence against signaling benefits playing a role in explaining the wage gap.

      Summary

      We’ve presented five factors that might lead economics majors to make more money than other liberal arts majors. Majoring in economics could increase employable skills, reflect employable skills, increase desire to make money, reflect desire to make money, and/or signal desirable traits to employers. The relative roles of these things has relevance to people who give substantial weight to their future earnings and are trying to choose between majors. In a future post, I’ll offer data and analysis that tests some of the hypotheses raised in this post.

      Appendix: Data showing that economics majors make more

      Here we report on survey data from seven sources, which consistently shows that economics majors make more money. The surveys sometimes have different methodologies and study different populations, so one should exercise caution in comparing numbers between surveys. Unless otherwise specified, numbers are in thousands of dollars. In some cases the sources did not have data on all of the majors that we considered: in such cases we listed the data that was available.

      The 2013-2014 Payscale Salary Report reports the following figures. The first column is major, followed by starting salary, midcareer salary, starting salary divided by economics major starting salary, and midcareer salary divided by economics major midcareer salary.


      Economics 50 96 1 1
      PoliSci 41 77 0.82 0.8
      Philosophy 39 78 0.78 0.81
      History 39 71 0.78 0.73
      English 40 71 0.8 0.73
      Psychology 36 60 0.72 0.62
      Sociology 37 55 0.74 0.57

      The National Association of Colleges and Employers Salary Survey gave 25th / 50th / 75th percentiles of starting salaries by major. In the last three columns, we compute these as percentages of the corresponding figures for economics.


      Economics 43 53 66 1 1 1
      PoliSci 33 40 50 0.76 0.75 0.75
      History 34 40 48 0.79 0.75 0.72
      English 32 38 48 0.74 0.71 0.72
      Psychology 30 35 43 0.69 0.66 0.65
      Sociology 29 35 43 0.67 0.66 0.65

      An PayScale report from 2008 gives more refined information about midcareer earnings by major: at the 10th, 25th, 50th, 75th and 90th percentiles


      Economics 50 70 98 145 210
      PoliSci 41 55 78 114 168
      Philosophy 35 52 81 127 168
      History 37 49 71 103 149
      English 33 44 64 93 133
      Psychology 31 42 60 87 127
      Sociology 30 40 58 81 118

      And here are the earnings as a fraction of earnings of economics majors at the corresponding percentile:


      Economics 1 1 1 1 1
      PoliSci 0.82 0.78 0.79 0.78 0.8
      Philosophy 0.7 0.74 0.82 0.87 0.8
      History 0.74 0.7 0.72 0.71 0.7
      English 0.66 0.62 0.65 0.64 0.63
      Psychology 0.62 0.6 0.61 0.6 0.6
      Sociology 0.6 0.57 0.59 0.55 0.56

      What’s It Worth: The Economic Value of College Majors gives 25th / 50th / 75th percentiles by given major (including both people just stating out and and people midcareer). The first three columns have these, and the last three have these as fractions of the corresponding economics major salaries.


      Economics 42 70 108 1 1 1
      PoliSci 39 59 90 0.92 0.84 0.83
      History 34 50 77 0.8 0.71 0.71
      English 34 49 71 0.8 0.7 0.65
      Psychology 31 45 65 0.73 0.64 0.6
      Sociology 33 45 68 0.78 0.64 0.62

      Heterogeneity in human capital investments: High school curriculum, college major, and careers (Table 1) gives hourly wage data by major using data from 2009, at the 10th percentile and 90th percentile, and at the 90th percentile excluding those with professional degrees, and comes up with the following. As above, the final three columns are salaries as fraction of economics majors’ salaries

      Economics 15.25 111.76 78.43 1 1 1
      PoliSci 14.71 75.41 57.25 0.96 0.67 0.72
      History 12.65 64.17 49.02 0.82 0.57 0.62
      Psychology 12.25 49.02 41.18 0.80 0.43 0.52
      At the low end, the differentials between the economics majors and others are lower than that reported in the 2008 Payscale report, and at the high end they’re greater. It’s unclear what’s going on here (presumably a difference in survey methodology), but regardless, economics majors come out on top.

      The Census Bureau’s 2010 American Community Survey fraction of people who majored in a given subject who are in the top 1% of Americans in earning power, by subject. Percentages are below:


      Economics 8.2
      PoliSci 6.2
      History 4.7
      Psychology 4.3
      English 3.8

      According to a Economics: Good Choice of Major for Future CEOs by Patricia Flynn and Michael Quinn:

      Economics ranked third with 9% of the CEOs of the S&P 500 companies in 2004 being undergraduate Economics majors, behind Business Administration and Engineering majors, each of which accounted for 20% of the CEOs. When adjusting for size of the pool of graduates, those with undergraduate degrees in Economics are shown to have had a greater likelihood of becoming an S&P 500 CEO than any other major.

      In Table 5, the authors give the probability of going on to be a CEO, as a fraction of the probability for an economics major:


      Economics 1
      PoliSci 0.31
      History 0.25
      English 0.08
      Sociology 0.05
      Psychology 0.04

      The value of the online hive mind

      The phrase “wisdom of crowds” was made popular in James Surowiecki’s eponymous book. The idea of aggregating a diverse range of opinions has been proposed in different forms, ranging from polling to prediction markets. Empirically, prediction markets perform somewhat better than crude polling, but just the act of aggregation itself improves significantly over not aggregating. Even crude aggregation mechanisms can be beneficial.

      Aggregation over larger numbers of people can be beneficial even if most people aren’t experts. However, it’s important to note that aggregation is beneficial only if enough people have at least a rudimentary knowledge of the subject, and those who don’t know anything are either unbiased or their biases cancel out(see The Myth of the Rational Voter for more). Aggregation with a certain level of filtering to sieve out the signal from the noise can overcome the problem of ignorance or even bias, as long as there is enough signal on the whole (i.e., enough people in absolute terms who know what they’re talking about).

      When you’re stuck with a question, whether personal, professional, or academic, it is often effective to turn to the hive mind for suggestions. Not that the hive mind can, or should, make your decisions for you. But it can offer valuable input that would otherwise take you a lot of time to collect.

      In the past, few people had access to the wisdom of the hive mind when it came to their own questions. Now, however, we have the Internet, and Internet research is a powerful way that people can access the hive mind for far more specific questions than they could have dreamed of before. There are many different types of onilne hive mind you could access:

      1. The Google/Internet hive mind: Search what the Internet as a whole has to say, using Google as your discovery tool. There’s a lot of wisdom out there. The advantage is that you can access a huge corpus of knowledge. The disadvantage is that you cannot ask your own questions and the knowledge isn’t arranged in a question-and-answer format.
      2. The Wikipedia hive mind: Avail of an “encyclopedia” that’s been written through the collaborative efforts of hundreds of thousands of people, and is regularly updated, to fill in the gaps in your knowledge and make an informed decision.
      3. The Quora/LessWrong/StackExchange/Reddit/discussion forum/blogosphere hive mind: Avail of stuff that’s explicitly designed for intellectual consumption, including stuff in the question-answer format. Also, ask your own questions and get answers (though not necessarily quickly).
      4. The Facebook(/Twitter?) hive mind: Ask quick questions and get quick answers from a select group of friends.

      Of these, (1) and (2) don’t rely much on your existing network of friends or followers. As long as your research skills are good, you can turn up the same material regardless of how good your friends and followers are at research. (3) involves a mix of research skills and the quality and size of your network of friends and followers. (4) is very heavily focused on the set of friends and followers you’ve accumulated.

      Is the hive mind actually helpful? To a large extent, this depends on how much the people involved know and/or have interesting things to say about the questions you pose to them. The narrower and more specialized your domain of inquiry, the more likely it is that the hive mind will not be any use. And for the Facebook hive mind (type (4) in my list), you need to have friends who have knowledge of the subject, check Facebook regularly, and are willing to comment. I now turn to my own experience.

      What have I used the hive mind for?

      The Google and Wikipedia hive minds are the ones I’ve used the longest, and they’re both indispensable to my process of discovery and research for the vast majority of subjects I try to learn about.

      I’ve used the Quora hive mind since I joined the site in June 2011, though my level of use has varied considerably.

      For other things that I’ve been interested in, either professionally or as a hobby, I’ve found the Facebook hive mind useful. This was not the case when I joined Facebook. It really started happening around late December 2012 and early January 2013, by which time I had accumulated a sufficiently large collection of Facebook friends who were (together) sufficiently widely knowledgeable and spent sufficient amount of time in total on Facebook. By “sufficient” here I mean “sufficient to make sure that enough of my posts attracted valuable comment feedback that I thought posting passed a cost-benefit analysis.” I’ve posted about a varied range of topics ranging from mathematics teaching to education in general to technological progress and social and political issues, and often learn a lot from the comments that I would probably either not have discovered by myself or have taken a much longer time to discover.

      However, these general-purpose hive minds are often not of much use for specific technical topics. I’ve also benefited from access to hive minds associated with more niche communities, some of them on Facebook or Quora, and others on their own websites or blogs. Back when I was working on my Ph.D. in group theory, the Facebook hive mind and Quora hive mind were little use for my research: less than a dozen of my friends knew enough group theory, and those who did didn’t check Facebook often enough. For the most part, I had to figure things out by myself, ask my advisor, or handpick individuals who would be likely to know. But I did have access to one hive mind, namely MathOverflow, that I used productively to ask many questions, one of which turned out to fill in an important gap in my thesis.

      How good are people at using these resources, and what advice is being offered to them?

      Let’s look at the four types of hive minds mentioned and how far people are from making use of them:

      1. The Google/Internet hive mind: There is a fair amount of research as well as commentary on how people use search engines for school work and other research. For instance, here’s a slideshare presentation from October 2010 (by these people) describing how people’s web research skills fall short and how they can be fixed. I’m not very confident of the quality of the advice offered, and also of its continued validity: much of it was written before some of the recent improvements in Google Search such as Google Instant and the knowledge graph (see this timeline of Google Search), and a lot of the advice doesn’t jive with my personal experience. But at any rate it’s a somewhat well-understood problem where people are actually trying to advise others on how to do it well rather than debating whether to do it at all.
      2. The Wikipedia hive mind: Effective use of Wikipedia has received a fair amount of attention. Wikipedia has its own page on Wikipedia research skills, including some cautionary notes about the particular issues with citing and using Wikipedia because of its role as an often-unvetted tertiary sources. There are also other articles and videos on the subject.
      3. The Quora/LessWrong/StackExchange/Reddit/discussion forum/blogosphere hive mind: These are relatively new, and “best practices” for these haven’t percolated to the people who write advice on study habits or general research skills. A biger problem is that a lot of people haven’t even heard of relevant websites like Quora, LessWrong, Stack Exchange, or the appropriate niche communities for them. So there’s some clear low-hanging fruit just in making them aware of the appropriate resources. That said, there are a few articles on effective use of Quora in particular, but these are largely in niche websites or the technology press rather than in stuff aimed at the general public. As described here, my experience with Cognito Mentoring advisees suggests that recommending to people to join Quora is one of the low-hanging fruit in terms of value we have been able to provide advisees.
      4. The Facebook(/Twitter?) hive mind: The problem here might be most severe, even though a fairly large number of people use Facebook and a reasonable number of people use Twitter. A fair number of people use Facebook as a hive mind for personal problems (such as opinions on a restaurant) but it’s not used for academic or research-related questions as much as it could be. Moreover, its use in this respect is generally not encouraged and not considered high-status. I’ll talk more about this in a subsequent post.

      I’m curious to learn about the personal experiences of LessWrong users on tapping into the online hive minds of various sorts, including categories that I’ve missed. In addition, views on how effectively most other people tap into the various online hive minds would also be much appreciated.

      Some pre-emptive remarks

      Pre-empting some criticisms I expect:

      • I don’t mean to imply that the only or even the primary purpose of websites such as Facebook is to answer one’s questions. Clearly, there are many other ways people derive value from the websites. This post is focused on the hive mind component of the value, and does not assert that that is or should be the most important reason for people to use Facebook.
      • The privacy issues surrounding websites such as Facebook and Quora are taken quite seriously by a number of people. I’m not trying to evaluate here whether the benefits of using these website exceeds the (perceived) privacy costs of doing so. I’m simply discussing one item that (I think) would go on the benefits side of the ledger.

      PS: Chris Hendrix comments on Facebook:

      It seems to me that there’s a logic of how to develop your various hivemind levels here. If you attempt to simply start with a FB group as your wisdom of the crowds you may not have enough knowledge to be able to determine whether or not your crowd selection is systematically biased in ways that don’t correlate with finding truth. Instead I think there’s a logic to building up each level of hivemind usage from the previous. From Google searches you will often be directed to Wikipedia. Wikipedia can then direct you to effective discussion sites (you hear about a discussion site, you check wikipedia to see if there are any criticisms of obvious failure modes). Finally, once you’ve found effective discussion sites, you’ve been learning what are useful and what are non-useful contributions. Since these sites will include a number of effective contributors you can pick and choose among this group to find people you can make into good facebook friends.

      I think done well, this can be a supplement (or perhaps even an alternative) to professional and academic networking for answering complex and non-obvious questions (the less complex and obvious ones are simply answered at the Google or Wikipedia levels normally).

      Cross-posted on LessWrong here and on Quora here

      The failed simulation effect and its implications for the optimization of extracurricular activities

      Cal Newport’s book How To Become a Straight-A Student The Unconventional Strategies Real College Students Use to Score High While Studying Less (that I blogged about recently) discusses a concept that Newport calls the failed simulation effect. Newport:

      The Failed-Simulation-Effect Hypothesis If you cannot mentally simulate the steps taken by a student to reach an accomplishment, you will experience a feeling of profound impressiveness.

      Newport, Cal (2010-07-20). How to Be a High School Superstar: A Revolutionary Plan to Get into College by Standing Out (Without Burning Out) (p. 182). Crown Publishing Group. Kindle Edition.

      Newport gives the following example in his book:

      Playing in a rock band doesn’t generate the Failed-Simulation Effect. You can easily simulate the steps required for that accomplishment: buy an instrument, take lessons, practice, brood, and so on. There’s no mystery. By contrast, publishing a bestselling book at the age of sixteen defies simulation. “How does a teenager get a book deal?” you ask in wonderment. This failure to simulate generates a sense of awed respect: “He must be something special.”

      Newport, Cal (2010-07-20). How to Be a High School Superstar: A Revolutionary Plan to Get into College by Standing Out (Without Burning Out) (pp. 182-183). Crown Publishing Group. Kindle Edition.

      On the basis of this insight, Newport’s bottom line for people looking for accomplishments in the high school extracurricular realm is:

      Pursue accomplishments that are hard to explain, not hard to do.

      My impression is that Newport is broadly correct as far as college admissions advice goes: activities that are hard to simulate seem more impressive, and therefore improve one’s chances at admission (ceteris paribus). But impressing admissions committees isn’t the only goal in life. In this post, I explore the question: how aligned is this advice to the other things that matter, namely, direct personal value (in the form of consumption and human capital) and social value?

      Understanding the question

      I’m interested in exploring how closely the following are related for a given accomplishment (with the exception of (1) and (2), the rest measure the value created in some respected; see also this page):

      1. Hardness: The amount of skill, effort, or character strength needed.
      2. Impressiveness (primarily in the context of signaling quality to colleges): The degree to which people (particularly college admissions officers) are impressed.
      3. Human capital: The useful knowledge and skills acquired in the context of the accomplishment.
      4. Social value: The benefit to the world from one’s accomplishment. Note that social value could be direct or indirect, mediated through later accomplishments that rely on the human capital and networking gains from the activity.
      5. Consumption: The fun or excitement of the accomplishment.
      6. Networking: Getting connected to people in the course of accomplishing.

      Newport’s insight is that hardness and impressiveness aren’t as closely correlated as we might want to believe. But how closely is impressiveness related to the items (3)-(6)? That’s what we want to explore here. But first, a bit about how hardness relates to the remaining items. For brevity, we will not discuss (5) and (6) further in the post. Instead, we will concentrate our energies on how (1) and (2) relate with (3) and (4).

      Where do hardness and impressiveness differ?

      Hardness and impressiveness aren’t completely uncorrelated. A few moments of introspection should reveal that that’s the case: if impressive things were easy to do, many people would be doing them, and they would cease to be impressive. But Newport’s central insight is that hardness and impressiveness aren’t as correlated as they seem on the surface. There are things that are quite hard to do but don’t seem impressive because they are mainstream and follow a standard path. There are other things that seem more impressive than their actual hardness warrants.

      Consider a 2 X 2 matrix

        Not impressive Impressive
      Not hard Not hard, not impressive (e.g., watching TV) Impressive but not hard (somewhat innovative activities, the sort that Newport wants to encourage more of)
      Hard Hard but not impressive (e.g., reaching ranked third in high school academics, learning a difficult musical instrument) Hard and impressive (e.g., becoming a really really good music player, getting a medal in a national math or sports contest)

      Note that the “hard but not impressive” characterization is relative. Being ranked third in high school academics is impressive. But it’s a lot less impressive to elite colleges (the colleges that Newport’s audience wants to get  into) relative to the amount of effort it takes to achieve. Similarly, learning a difficult musical instrument is somewhat impressive, but not as impressive as it is hard.

      We restrict attention in this post to hard activities that people seriously consider doing, rather than random hard stuff people may do for dares or bets (like staying up for 100 hours at a stretch).

      Newport wants to shift people from the “not impressive” column to the “impressive” column, and notes that there are plenty of activities in the top right quadrant.

      What qualitative attributes characterize activities in the top right quadrant, and their very opposite, namely, activities in the bottom left quadrant? Some observations (based on Newport):

      1. Standard versus nonstandard: Activities that a lot of people are already doing don’t seem that impressive, even if they are hard. And some relatively hard activities are a standard part of people’s academic and extracurricular experience. Learning AP BC calculus and writing a rudimentary mobile app may be of roughly equal hardness. But a lot of people are doing the former since it is part of the standard path. Learning how to play the violin may be about as hard as doing research in a marine biology lab. But the former is a relatively standard extracurricular activity that many people do because it’s what they are supposed to do, or because their parents force them to do it.
      2. Outward-facing: Things that seem like they serve larger numbers of people seem impressive. Mastering one’s learning of a subject is less impressive than doing something that reaches out to many people. But this could have more to do with the “Convincing people” point I make below.
      3. Convincing people: Activities that involve changing other people’s minds seem prima facie more impressive. Learning the violin doesn’t require convincing anybody of anything. You just sit down and learn (or take lessons). Getting somebody to publish your book, on the other hand, requires convincing a publisher that your book is worth publishing. And getting people to buy the book requires convincing buyers that the book is worth buying. Creating an online community or successful marketing/lobbying for a nonprofit both have the flavor of convincing people. It makes sense that having convinced people is a convenient indicator of having done something impressive. In a sense, the evaluation of impressiveness has been outsourced to the other people already convinced.
      4. Discrete original projects: I’m not too sure of this, but people do seem to be somewhat biased in favor of discrete, distinctive projects with clearly identified names or a distinctiveness of identity. “I created a popular website with 1000 pages of information about topic X” sounds more impressive than “I wrote 1000 Quora answers about X” even if it’s the case that the latter activity generates more pageviews in the long run.

      Hardness, impressiveness, and human capital

      Now that we’ve identified some general points of divergence between hardness and impressiveness, we can consider the question: how do hardness and impressiveness differ in terms of the extent to which they correlate with human capital acquisition (i.e., the acquisition of knowledge and skills that have long-term utility)? As before, we restrict attention to hard activities that people seriously consider doing, rather than random hard stuff people may do for dares or bets (like staying up for 100 hours at a stretch). Let’s look at the four potential sources of divergence and compare based on those:

      1. Standard versus nonstandard: The “hard but not impressive” cluster comprises the more standard activities, whereas the “not hard but impressive” cluster comprises more nonstandard activities. So, this consideration boils down the question to: do standard activities produce more human capital than nonstandard activities? My answer is (very guardedly) mildly in favor of standard activities. Although much of school learning is wasteful, the standard subjects still have the benefit of several years of curriculum development that provides a certain bare minimum of quality. Nonstandard stuff exhibits higher variance. I suspect that the typical nonstandard activity is worse for building human capital than the typical standard activity. But I also think there’s more scope for doing really well on the human capital end by picking a really good nonstandard activity. Another consideration in favor of nonstandard is that there’s a large supply of people who can do the standard stuff, so that the marginal value of adding another person with standard skills is high, whereas the nonstandard stuff could involve building rare, specialized skills.
      2. Outward-facing: My guess is that at the high school level, the most high-value activities (from the human capital perspective) tend to involve learning about the world (not limited to what’s in school syllabi) rather than creating products. This isn’t a hard-and-fast rule, but it does point in the direction of impressive activities being less valuable from the human capital perspective than hard activities.
      3. Convincing people: This argues in favor of impressiveness. The skill of convincing people is an important one, and the act of convincing people also requires one to do a better job overall with presentation and background knowledge. This is good preparation for later life, where one needs to often suggest new things and convince people of them.
      4. Discrete original projects: Impressiveness favors discrete original projects. I think this is an argument in favor of impressiveness being better at building human capital, but a very weak one. People acquire valuable skills in the process of creating their own original projects that they wouldn’t when contributing to existing projects (for instance, creating your own website means you have to learn about website creation and getting traffic). On the other hand, participating in existing projects makes it easier to calibrate your learning, get feedback, and improve.

      Hardness, impressiveness, and (direct) social value

      How do the “impressive but not hard” activities compare with the “hard but not impressive” activities in terms of the
      direct value they produce for society? We’ll do a point-by-point comparison similar to that for human capital, but first, a little digression.

      Although many hard activities are not valuable, it is almost always the case that valuable activities are at least somewhat hard. The logic is similar to the logic for hard and impressive activities described earlier in the post. Namely, if valuable activities were easy to do, they would already have been done to the extent where they either became hard at the margin or lost value at the margin.

      PayPal co-founder Max Levchin credits this insight to co-founder Peter Thiel (see here). Levchin recounts that, back when PayPal was in its infancy, he was enamored by the idea of using elliptic curve cryptography to speed up some aspects of PayPal’s secure transactions. Elliptic curve cryptography uses some pretty cool math and offers interesting implementation challenges. But it turned out that the speedup offered wasn’t really helpful with the things that PayPal needed to do. Levchin learned from Thiel that hardness isn’t the source of value. On the other hand, things that are valuable are almost always bound to be hard, because if they were easy, they’d have already been accomplished. Indeed, Levchin’s new company, named Hard Valuable Fun, builds on this insight.

      As before, we restrict attention to hard activities that people seriously consider doing, rather than random hard stuff people may do for dares or bets (like staying up for 100 hours at a stretch). Now, let’s compare hard and impressive activities in terms of their social value:

      1. Standard versus nonstandard: The “hard but not impressive” cluster comprises the more standard activities, whereas the “not hard but impressive” cluster comprises more nonstandard activities. So, this consideration boils down the question to: do standard activities produce more direct social value than nonstandard activities? I think the general answer is a resounding no. Standard activities are largely focused on building human capital or signaling quality (to colleges and others), rather than on the creation of direct social value. This is true even for standard extracurriculars, such as learning musical instruments. Even the standard extracurriculars billed as socially useful, such as volunteer work by US students in Columbia, often produce negligible social value (see Jonah’s post on volunteering for a more in-depth discussion). Note that the indirect social value created through human capital acquisition might still be huge for some activities that build human capital, but that is not what we’re trying to assess in this part of the post. Nonstandard activities exhibit higher variance, but could at least in principle be chosen for higher social value. Another consideration in favor of nonstandard is that there’s a large supply of people who can do the standard stuff, so that the marginal value of getting something nonstandard accomplished may be higher on account of more low-hanging fruit.
      2. Outward-facing: Impressive activities tend to be outward-facing. And creating direct social value generally requires being at least somewhat outward-facing. So, this consideration points in favor of impressiveness over hardness.
      3. Convincing people: This argues in favor of impressiveness. Creating positive change usually requires convincing people at some level. This could be direct suasion, or it could be attracting people to visit one’s website or buy one’s book or use one’s products in some other capacity.
      4. Discrete original projects: Impressiveness favors discrete original projects. I think this is an argument in favor of impressiveness being better at creating direct social value, but a very weak one, and there are many counterexamples. Creating your own website may seem more impressive than just writing a bunch of Quora answers, but the latter may get read a lot more.

      Below, I summarize what I’ve said about hardness, impressiveness, human capital, and direct social value:

      Consideration Human capital consideration points in favor of hardness or impressiveness? Direct social value consideration points in favor of hardness or impressiveness?
      Standard versus nonstandard Hardness (but weak) Impressiveness (but weak)
      Outward-facing Hardness (but weak) Impressiveness
      Convincing people Impressiveness Impressiveness
      Discrete original projects Impressiveness (but very weak)

      Impressiveness (but very weak)

      Overall, it seems that a shift towards impressiveness would perform better in terms of direct social value and slightly worse in terms of human capital. But the variation between different choices of activities overwhelms the general comparison of hardness and impressiveness. In other words, there are probably a lot of activities within the impressive category (at varying levels of hardness) that perform well on the human capital and direct social value dimensions. One just needs to be know to look for them.

      Any thoughts on the above would be appreciated.

      PS: I’m planning to do another post (or posts) on how people in high school and early college, or others in a similar age group, can select side projects and execute them well.

      Cross-posted at LessWrong and Quora

      How much does where you go to college affect earnings?

      Students who go to more selective colleges make more money later in life. The 2013-2014 PayScale College Salary Report gives starting salaries of ~$60k for graduates of Ivy League schools vs. ~$45k/year for graduates of mid-tier state schools, and mid-career salaries of ~$110k/year for graduates of Ivy League schools vs. ~$80k/year for mid-tier state schools, so the difference is about 30%.

      This partially reflects students who go to more selective colleges being more able and ambitious, as opposed to attending a more selective college boosting one’s income. How much does attending boost income? The famous paper Estimating the Return to College Selectivity Over the Career Using Administrative Earning Data (2011) by Dale and Krueger raises the possibility that on average, attending a more selective college doesn’t raise earnings at all. Specifically, controlling for

      • Student high school GPA
      • Student SAT score
      • The average SAT score of the colleges that a student applied for1
      • The number of applications that the student submitted
      • Various demographic factors (race, sex, parental education)

      they found that as a group, there was no statistically significant difference in income later in life between students who went to more selective colleges and students who went to less selective colleges. Their finding is somewhat robust: it’s based on a large (~10k) sample size, it’s true both of the class of 1976 and the class of 1989, it’s true of the class of 1976 from age 25 through age 50 and it’s true both of men and of women.2,3

      A couple of caveats:

      • Blacks, Hispanics, and children of parents who don’t have college degrees and who attended more selective schools earned more than those who did not.
      • The 27 universities that the students were drawn from are highly selective: if one were to look at less selective universities, one might get different results.

      Dale and Krueger’s central finding has been taken to be evidence that going to a more selective college does not increase one’s earnings, contrary to conventional wisdom.

      There are many factors that could give rise to a difference in income between those who attended more selective colleges, some of which favor those who attended less selective colleges, and could counterbalance others that increase earnings. I list some potential contributing factors below (some of which were pointed out by Dale and Krueger). In each case, whether these effects are present is unclear, and to the extent that they are there, the overall effect of them is sometimes unclear, but they could give rise to a difference between the two groups.

      Selection effects

      These have no bearing on whether going to a more selective college increases expected earnings.

      Cost

      The cost of attending a more selective college could (after taking a student’s ability and merit-based financial aid into account) be higher or lower than the cost of attending a selective college. So the choice of attending a more selective school could reflect having a family that’s willing to pay more for college (and/or the willingness of the student for his or her family to pay).

      Future career plans

      Students who choose to attend more selective schools could be more or less likely to go into academia than students who don’t. Academics make less money than other people do (after controlling for factors such as GPA, SAT scores and conscientiousness). Similarly, those who attend more selective schools may have other career preferences that feed into expected earnings

      For students who go on to professional school (law, medicine or business), college attended doesn’t matter as much for later life prospects. So attending a more selective college could reflect lower intent to go on to professional school. The earnings of people who go onto professional school are generally higher than those of people who don’t – this points in the direction of expected earnings being higher for those who go to less selective colleges.

      Concern for prestige

      Students who choose to attend more selective colleges plausibly care more about prestige. This desire for prestige could correspond to more desire to make money later in life – this points in the direction of expected earnings being higher for those who go to more selective colleges.

      Signaling

      The impact of prestige of college attended on hiring decisions

      According to research by Lauren Rivera, high paying elite professional service firms (investment banks, law firms, and management consulting firms) give a lot of weight to whether job applicants attended top four universities when making their hiring decisions.

      More broadly, employers give weight to the prestige of college attended. However, the effect size is smaller than it might seem. In a 2013 Gallup Poll, 9% of business leaders said that the college a job applicant attended is “very important” to managers making hiring decisions, and 37% said that it’s “somewhat important.” Notably,

      (i) employers listed college attended as the least important of the 4 factors that they were asked about

      (ii) the American public gives more credence to employers weighting college attended when making hiring decisions: 30% think it’s “very important” and 50% think it’s “somewhat important.”

      (The 2013 data is much more recent than the Dale-Krueger data, but still provides relevant evidence.)

      Once one is hired, where one went to college won’t play a role in professional advancement at that firm (except to the extent that it built relevant skills).

      Influence on grades

      Carl Shulman suggested that going to a more selective school reduces one’s expected GPA, because of higher grading standards. It’s unclear to me whether this is true, and the effect could even cut the other way, but there’s plausibly an effect in some direction. Reduced GPA reduces one’s prospects for getting into medical or law school. Perusing forums for applicants, one finds many people saying that GPA is a dominant factor in admissions, one that’s far more important than prestige of college attended.

      Influence on major choice

      Attending a more selective school reduces one’s relative standing amongst students in a given major. This can lead to students at more selective schools choosing a less demanding major than they otherwise would have (whether because the major requires more effort than it would have, because one finds it intolerable to be one of the weaker students majoring in a given subject, or for some other reason). In the 2013 Gallup Poll cited above, business leaders surveyed listed the subject that a student majored in as significantly more important than college attended, in the context of hiring decisions.

      Treatment effects

      Peer group

      Having a more capable peer group can lead to better learning opportunities, and higher expected earnings. Ben Kuhnwrote

      By watching how more competent people work and think, you can often pick up useful study habits and better techniques for the subject you’re studying. I’ve found this especially true in CS classes, where I’ve had this experience from both sides, e.g. teaching classmates how to use Git and picking up C coding style and tricks from better programmers.

      It can also give one access to better advice: Ben Kuhn also wrote:

      Knowing talented students has given me info about several excellent courses, as well as summer opportunities, I wouldn’t otherwise have known about.

      These considerations generally favor more selective schools, but not as strongly as might meet the eye: less selective schools often have honors courses and honors programs, where one might be able to meet students as capable as those who one would be interacting with at less selective colleges (though the best students at more selective colleges will generally be stronger than the best students at less selective colleges).

      Networking benefits

      Going to a more selective college will generally expose one to people who will be in higher places later on in life, and who will correspondingly be able to connect one with influential people in one’s professional field, who may get one a high paying job and so forth. Such people may also serve as professional collaborators, for example, if one wants to do a startup right out of college.

      As above, the effect here is smaller than might initially meet the eye, because one might be able to get similar benefits by interacting with the most capable students at a less selective college.

      Confidence

      Some people have suggested that being a “small fish in a big pond” reduces students’ confidence relative to being a “big fish in a small pond.” Assuming this, to the extent that confidence increase later life earnings, all else being equal, attending a more selective school will reduce expected earnings.

      Better learning due to attention from professors

      Some people have found that being a “big fish in a small pond” is conducive to getting more attention from the professors in one’s classes, on account of standing out. This can increase the amount that a student learns, because of professors’ greater willingness to spend time on personalized instruction.

      Self-sufficiency

      Some people who I know report to having a subjective sense that being in a less elite environment was helpful to them, because it forced them to be independent (on account of being different from their peers), whereas had they been in an elite environment, they would have “gone with the flow” and uncritically made their decisions based on what their peers were doing. All else being equal, this factor would increase expected earnings of those who attend less selective colleges.

      Influence on major choice

      As above, attending a more selective college can reduce the probability that one will major in a demanding subject. Aside from having signaling implications, this also reduces the chances that one will acquire technical skills required for higher paying jobs.

      Influence on career choice

      Going to a more selective college could nudge one toward or away from academia, going into the non-profit world, getting a professional degree, etc. Earnings vary across these fields, and this could give rise to a difference between the groups.

      Implications

      Before thinking seriously about the paper by Dale-Krueger, I had subscribed to the conventional wisdom that going to a more selective college generally boosts expected earnings. Thinking it over more carefully, it’s now genuinely unclear to me whether this is the case, and it could reduce expected earnings in general.

      Rather than taking the Dale-Krueger finding to be definitive, one should give some weight to conventional wisdom, on the grounds that the paper might have hidden methodological errors, or have ceased to be relevant in the present day. But in view of

      1. The Dale-Krueger finding
      2. The fact that there are a number of ways in which going to a more selective college could decrease earnings
      3. The fact that and the fact that the salient advantages of going to a more selective college are less significant than they might initially appear
      4. The fact that conventional wisdom is at least partially rooted in a conflation of correlation and causation

      it seems reasonable to adopt a “best guess” that if going to a more selective college does increase expected earnings, the effect size isn’t high (perhaps on the order of ~2% – an intuitive guess based on the data given in the Dale-Krueger paper).

      What implications does this have for students who are trying to decide what college to go to, or how much to focus on getting into college? First some general considerations:

      • Rather than taking the Dale-Krueger finding to be definitive, one should give some weight to conventional wisdom, on the grounds that the paper might have hidden methodological errors, or have ceased to be relevant in the present day. But in view of (i) the Dale-Krueger finding (ii) the fact that there are many ways in which going to a more selective college could decrease earnings (iii) the fact that and the fact that conventional wisdom is at least partially rooted in a conflation of correlation and causation, it seems reasonable to adopt a “best guess” that if going to a more selective college does increase expected earnings, the effect size isn’t high (perhaps on the order of ~2% – this is an intuitive guess based on the data given in the Dale-Krueger paper).
      • Even if the causal effects lead to there being no difference in expected earnings on average and at the moment, that doesn’t mean that there’s no difference for a rational actor who wants to maximize expected earnings. For example, if attending a more selective (or less selective) college were to reduce expected earnings on account of increasing the probability that students go into academia, if one doesn’t want one’s earnings to be reduced, one can simply buck the trend and not go into academia. However, there are certain casual effects over which one doesn’t have control (such as the fact that elite firms look more favorably on students who graduated from top 4 colleges)
      • The casual effects will vary from person to person. For example, going to a college that elite finance firms look on favorably will have an effect for those who aspire to go into finance to a greater degree that it won’t for those who don’t aspire to go into finance. A high school student can try to assess which causal effects will apply to him or her. But it can be hard to tell ahead of time: before entering college, one might be unclear on (or mistake about) whether one wants to go into finance.
      • Expected earnings is not the only metric of the value of going to college. There’s also the consumptive experience, as well as well as its impact on career success that’s not reflected in earnings. It’s plausible that going to a more selective college matters more for professional success for people who will be going on to academia than it does in general.
      Irrespective of the considerations above:
      • The Dale-Krueger finding generally argues points in the direction of giving greater weight to cost differences between colleges than to differences in prestige. Depending on one’s family’s income and savings, going to Harvard can be cheaper than going to Berkeley, but for other families there will be a difference between the two on the order of $100k (which if invested for 30 years would grow to ~$800k).
      • The Dale-Krueger finding points in the direction of effort spent getting into college being less cost-effective than most people think, which should shift one in the direction of spending less effort on it and more effort on building employable skills, enjoying life, and contributing social value.

      Footnotes

      [1] You might wonder why the authors didn’t control for the average SAT score of the colleges at which the students were accepted. The authors did something like this (actually, comparing students who had been accepted at the exact same set of colleges) in a 2002 paper, and obtained similar results. From the paper: “The matched applicant model and self-revelation model yielded coefficients that were similar in size, but the self-revelation model yielded smaller standard errors. Because of the smaller sample size in the present analysis, we therefore focus on the self-revelation model.”

      [2] The authors say this in the text of the paper, but when I look at Table 4 of the paper on page 32 of the PDF, the data seems to indicate that for women from the 1976 cohort broken down into age groups, there is a statistically significant difference, in favor of those who attended less selective schools. But I assume that I’m misinterpreting the table: I’d welcome any help interpreting the data on this point.

      [3] The authors qualify this by saying “The estimates from the selection-adjusted models are imprecise, especially for the 1989 cohort. Thus, even though the point-estimates for the return to school quality are close to zero, the upper-bound of the 95 percent confidence intervals for these estimates are sometimes sizeable.”

      A summary and broad points of agreement and disagreement with Cal Newport’s book on high school extracurriculars

      Cal Newport (personal website, Wikipedia page) is a moderately well-known author of four books as well as a computer science researcher. I have read two of his four books: How To Become a Straight-A Student The Unconventional Strategies Real College Students Use to Score High While Studying Less and How to Be a High School Superstar: A Revolutionary Plan to Get into College by Standing Out (Without Burning Out). I’m particularly interested in his book on becoming a high school superstar. My interest arises as part of trying to figure out how people can better use their extracurricular activities to have more fun, learn more, and create more value for the world. As Jonah recently pointed out, choosing high school extracurricular activities could in principle have huge social value in addition to the private benefits. And as far as I know, Cal Newport is the only person who has given systematic advice on high school extracurriculars to a broad audience. He’s been referenced many times on Less Wrong.

      In this post, I’ll briefly discuss his suggestions in the latter book and some of my broad philosophical disagreements. I’m eager to know about the experiences of people who’ve tried to implement Newport’s advice (particularly that pertaining to extracurriculars, but also any of his other advice). First impressions of people who click through the links and read about Newport right now would also be appreciated. I intend to write on some of these issues in more detail in the coming days, though those later posts of mine will not be focused solely on what Newport has to say.

      You might also be interested in the comments on this Facebook post of mine discussing Newport’s ideas.

      A quick summary of Newport’s views

      Newport’s book advises high school students to pick an extracurricular activity and shine at it to the level that it impresses admissions officers (and others). He offers a three-step plan for highschoolers:

      1. The Law of Underscheduling: Pack your schedule with free time. Use this free time to explore: In particular, avoid getting being involved in too many activities, whether academic or extracurricular. Use your free time to read and learn about a wide range of stuff.
      2. The Law of Focus: Master one serious interest. Don’t waste time on unrelated activities: Newport cites the superstar effect and the Matthew effect to bolster his case for focusing on one activity after you’ve explored a reasonable amount.
      3. The Law of Innovation: Pursue accomplishments that are hard to explain, not hard to do: Newport talked of a “failed-simulation effect” where things seem impressive if the people who hear about them can’t easily imagine a standard path to them. He then offers some more guidelines both on how to innovate and on how to make one’s innovation seem impressive.

      Newport is targeting high school students who want to get into their dream college. He’s trying to get them to stop doing boring, depressing activities and instead do fun, creative, and useful stuff that both improves their short-run life (by making them more relaxed and less stressed) and impresses admissions officers.

      Broad areas of agreement

      1. I think Newport is right to suggest that it doesn’t make sense to devote too much energy to boring schoolwork or extracurriculars that one is doing just because one is “supposed” to do them. I think he’s right that his approach is both less stressful and less wasteful of human resources and effort. And it is more likely, in expectation, to build human capital and produce direct value for society.
      2. Newport is correct to emphasize the link between free time and being able to explore stuff, and his advice on how to explore can be quite helpful to high school students.
      3. Newport’s ideas for how to focus on a particular interest, and how to rack up accomplishments in a particular area, seem broadly sound.
      4. When it comes to figuring out what impresses college admissions officers, Newport seems like he knows what he’s talking about, although some of his examples make less sense than he thinks they do.

      Broad philosophical differences

      Before getting into the nuts and bolts of what I think Newport gets right and wrong, I want to talk of some broad differences between Newport (as he presents himself) and me. A few things I find somewhat jarring in Newport’s writing:

      1. Newport seems very concerned with signaling quality to colleges. This is fine: that’s what his target audience cares most about, and if getting into a good college is important, then signaling quality to college can be quite important. What I find somewhat offputting is that he often confuses the signaling with the value of the activity itself, or at any rate fails to question whether some of the things he believes to be optimal from the signaling viewpoint could be counterproductive from the perspective of value creation (either personal or social). For instance, consider his observation of the existence of the failed-simulation effect. This points in favor both of picking things that are harder for other people to “see through” (rather than things that are straightforward but hard) and also in favor of making what you did seem more undoable than it actually is. I see these as downsides of the failed-simulation effect, and sources of genuine conflict between choosing what creates the most value (personal or social) and what impresses others. Newport seems to sidestep such dilemmas.
      2. Newport doesn’t adequately address the zero-sum context in which he is giving his advice. Top colleges have a limited number of places for students. If everybody successfully implemented Newport’s advice, only a small fraction of them would be able to go to a top college. Note that I don’t think Newport views his advice as zero-sum, and even if what I wrote above is correct, his advice could still be positive-sum in that it shifts people away from competing on stressful dimensions to doing activities that offer them more fun and learning and create more value. But again, the fact that he doesn’t really address this issue head-on is a disappointment.
      3. Newport seems to oversystematize in ways that don’t feel right to me. Even though I agree with aspects of the broad direction he is pushing people in, I feel he’s seeing too many patterns that may not exist.
      4. In general, I feel that Newport doesn’t go far enough. He operates within the standard set of constraints without questioning the logic of the enterprise or giving people a better understanding of the incentives of different actors in the system. He also doesn’t provide adequate guidance on the self-calibration problem, and doesn’t adequately encourage people to figure out how to calibrate their learning better in the context of the extracurricular activity where they cannot rely on standard measures such as grades to track their progress.

      I’m curious to know what readers’ main areas of disagreement with Newport are, and/or whether my listed areas of disagreement make sense to readers.

      Cross-posted to LessWrong and Quora.

      Career prospects for physics majors

      Physics is attractive to many highly intellectually capable students, because

      • Physical theories represent pinnacles of human achievement
      • It’s intellectually stimulating
      • It has a reputation for being a subject that smart people do

      See the comments on the post What attracts smart and curious young people to physics?

      But what of career prospects?

      In an answer to the Quora question What is it like to major in physics? PhD physicist Joshua Parks wrote:

      It may not be too crazy to claim that as far as career options go, physics majors may be much more like English or other humanities majors (who often make career choices unrelated to their study) than their science and engineering counterparts.

      At Physics Forums, ParticleGrl wrote

      If you are an engineer, you can almost certainly get a job in a technical field right out of college. Physics majors, on the other hand, end up all over the place (insurance, finance, teaching high school, programming, etc).

      We discuss some career paths for physics majors below.

      Summary

      • The primary reason to major in physics (outside of intrinsic interest) is as a prerequisite to a physics PhD or as background for teaching high school physics.
      • Over 50% of those who get PhDs in physics don’t become physicists, often because of difficulty finding jobs.
      • Physics majors are able to get jobs in other quantitative fields, but often with more difficulty than they would had they majored in those fields.

      The popularity of physics as a major

      The fraction of students who major in physics is small. What’s It Worth?: The Economic Value of College Majors by the Georgetown University Center on Education and the Workforce (pg. 162) reports that there are 936k people with Bachelor’s degrees in physical sciences, but only 91k with degrees in physics. Assuming that there are 50 years worth of people with Bachelor’s degrees in the United States, we get figures of about 20k physical sciences majors per year and 2k physics majors per year. This is in consonance with areport of the National Center for Educational Statistics, which gives a figure of 20k physical science majors who graduated in 2005. There are about 1.3 million college majors a year, so on the order of 0.2% of college graduates majored in physics.

      The proportion increases significantly if one considers the population of highly intellectually capable students. For example, about 2% of Stanford undergraduates major in physics. The proportion will be still higher if one considers the population of Stanford’s most intellectually capable students.

      Physics

      The Bureau of Labor Statistics reports that 17k people work as physicists, so about 20% of physics majors.

      Majoring in physics is a step toward becoming a physicist, but it’s usually not sufficient. In an anonymous answer to the Quora quest What is it like to major in physics?, the answerer reports

      There are no jobs in physics as the BS level. You need a PhD to do work related to physics, and even work at the Masters level is not that great (so I’ve heard).

      This may not literally be true: the American Physical Society reports that 5% of physics majors who enter the workforce right after college work in physics or astronomy. But broadly, a physics PhD seems to be a prerequisite to becoming a physicist.

      Graduate school is a common path for physics majors. What’s It Worth? reports (pg. 27) that 67% of physics majors go on to earn a graduate degree (without giving a breakdown of what kinds of graduate degrees they get). The American Institute of Physics reportsthat there are about 900 US citizens who earn physics PhDs a year, suggesting that a large fraction (30+%) of the ~2k physics majors who graduate in a given year go on to earn PhDs in physics.

      The default career path for a PhD physicist is academia. We give some general considerations on our page on Academia as a career option. There seems to be a general consensus that the job market in physics academia is extremely competitive. Don’t Become a Scientist! by Jonathan Katz describes the scarcity of jobs relative to PhDs and its implications. Physicist rknop writes

      My own field is physics, and the problem of physicists being trained for and expected to get tenure-track faculty positions, without enough of these positions being out there, has been a sore topic for two decades (at least). […] There is absolutely no guarantee that the PhD will allow them to spend the rest of their lives in physics research.

      Putting the number of physicists together with the number of physics PhDs, it appears as though roughly 50% of physics PhDs are physicists (whether in academia or industry).

      Success in physics seems to be driven in large part by intelligence, so exceptionally intelligent people may have an easy time getting a job, but they have to be sufficiently intelligent to stand out amongst a population that’s already strongly selected for intelligence.

      Computer programming / software engineering

      What’s It Worth? (pg. 165) reports that 19% of physics majors end up in “computer services.” This is vague, but it seems reasonable to guess that it’s mostly software engineering. Answers to the Quora question Why are there so many physics majors in software engineering? give some reasons for this.

      Physics majors’ coursework and research can involve computer programming, but this tends to be limited. Broadly, if one wants to be a software engineer as a physics major, one has to minor in or double major in computer science, or spend a significant amount of time programming on one’s own. In general, one can get a job as a software engineer without a computer science degree, so majoring in physics exclusively doesn’t bar one from the career path, but it also seems strictly inferior to majoring in computer science from a professional point of view, for future software engineers.

      In an answer to Can a physics major get hired as a software engineer? at Physics Forums, fss writes

      You will start out at a disadvantage compared to computer science people who have demonstrated programming ability, and it will be up to you to decide how best to show that you can bring something to the table that would make up for this deficiency (real or perceived).

      Engineering

      The American Physical Society reports that 32% of physics majors who enter the workforce directly go into engineering. What’s It Worth? (pg. 165) reports that 17% of physics majors are engineers.

      The answers to Can a Physics major get a job as an engineer? and Engineering Job with a Physics Degree at Physics Forum suggest that physics majors can get jobs as engineers, but that they’re at a disadvantage relative to engineering majors, and that those who plan to be engineers should major in engineering.

      Physics majors are sometimes able to go to engineering graduate school, for example, Dan Recht.

      High school teaching

      The Physics Teacher Education Coalition reports that there are 27k high school physics teachers, 35% of whom have degrees in physics or physics education, suggesting that up to 10% of physics majors become high school physics teachers. We have not yet done a writeup on high school teaching as a career, but hope to do so.

      Earnings

      • Payscale reports that median midcareer salary for physics majors is $101k/year, which ranks 9th in median midcareer salary amongst majors, after computer science, actuarial mathematics, and some engineering specialties.
      • The median starting salary for physics majors of $53k/year is lower than the median starting salary for engineers, which is more like $60k-$65k/year.
      • What’s It Worth? (pgs. 23-24) reports that the 25th percentile of physics majors’ income is $38k/year, compared with $85k/year for engineering specialties.

      The relatively low median starting salary and 25th percentile salary may be dragged down substantially by the fact that physics majors attend graduate school and do postdocs with higher frequency than engineering majors do, during which they have low earnings.

      After controlling for years of education and intelligence, physics majors make less than engineers, even mid-career. As above, physics majors complete PhDs more frequently than engineering majors do, and one source reports that physics majors’ average SAT scores are about 100 points higher than engineering and computer science majors’ on a 1600 point scale (equating to about 0.5 SD in IQ). So it’s plausible that they make less money than their counterparts of similar intelligence who majored in engineering or computer science. This doesn’t necessarily mean that they couldn’t get jobs where they made more money – it could be that they prefer lower paying academic jobs over higher paying jobs outside of physics.

      Biomedical research as a career

      I did some preliminary research on biomedical research as a career. The case for becoming a biomedical researcher looks to be weak for most candidates for the career. Are there important points in favor of pursuing a career in biomedical research that I’m missing?

      Summary

      • Some people find biomedical research very rewarding, but the job involves a lot of grant writing, not only research.
      • Job security for biomedical researchers in academia is extremely poor before tenure. We still have to research exit options for those who leave academia.
      • Biomedical researchers make substantially less money over a life time than they could in other fields.
      • The job involves ~60 hours of work per week
      • While biomedical research has historically produced a great deal of value, the situation today is more ambiguous, and it appears that the average biomedical researcher does little to advance the field.

       

      The nature of the work

      According to How to succeed in science: a concise guide for young biomedical scientists. Part I: taking the plunge by Yewdell (2009)

      for individuals with a hunger for knowledge and an insatiable curiosity about how things work, science offers a constant challenge and, best of all, the intense thrill of discovery.  What can match being the first person who has ever lived to know something new about nature? And not just the big, infrequent, paradigm-making (or breaking) discoveries, but the small, incremental discoveries that occur on a daily or weekly basis too. If this doesn’t give you goosebumps and if you are not in a rush to get to the laboratory in the morning to find the results of yesterday’s experiment, then you should seriously consider a non-laboratory career.

      However, research is not the only part of the job: Yewdell writes

      For your entire career as a PI, you will put inordinate efforts into writing grants

      This is in consonance with GiveWell’s post Exploring Life Science Funding which says

      The existing system focuses on time-consuming, paperwork-heavy grant applications for individual investigators.

      GiveWell’s post also hints at researchers being constrained with respect to the research that they’re able to get funding for:

      The existing system favors a particular brand of research – generally incremental testing of particular hypotheses – and is less suited to supporting research that doesn’t fit into this mold. Research that doesn’t fit into this mold may include: (i) Very high-risk research representing a small chance of a big breakthrough. (ii) Research that focuses on developing improved tools and techniques (for example, better microscopy or better genome sequencing), rather than on directly investigating particular hypotheses. (iii) “Translational research” aiming to improve the transition between basic scientific discoveries and clinical applications, and not focused on traditionally “academic” topics (for example, research focusing on predicting drug toxicity).

      Job security

      Our writeup on job security in academia gives some general considerations.

      Concerning biomedical research specifically, The Scientific Workforce Policy Debate: Do We Produce too Many Biomedical Trainees? reports that

      During the period from 1993-2003, the probability that a postdoc in the U.S. was in a tenure-track PI position 5-6 years after obtaining their PhD ranged from 15-23% (Garrison and McGuire, 2007).

      This graphic says that after finishing graduate school / postdoc, of biomedical research PhDs, 18% go into non-research science jobs, 6% go into government research, 43% go into academia or teaching, 18% go into industrial research, 13% do work outside of science and 2% are unemployed. Roughly 50% of those who complete a postdoc and go into academia get tenure, and the career outcomes for those who don’t get tenure are unreported.

      Some of the jobs that biomedical researchers get outside of academia are jobs that they could have gotten without doing a PhD or postdoc.

      An important question is that of how correlated research ability is with job security. If luck plays a sufficiently large role then high ability doesn’t guarantee a job, whereas if skill can overcome luck, then those who are skilled can be confident that they’ll be able to get jobs. An interview with Prof. Andrew McMichael at the 80K blog seems to suggest that sufficiently high quality researchers can get jobs and funding. However, going into graduate school, one’s ability level may not be clear.

      It’s unclear how job security is changing over time. In 2010, the Bureau of Labor Statistics reported that the number of jobs was expected to grow 36% over 10 years (much faster than average). But in 2012, the Bureau of Labor Statistics reportedthat the number of jobs is expected to grow 13% over 10 years, and in the intervening time the number of jobs had grown only 3%. So there appears to have been a substantial change in outlook in only two years. The job growth rate forecasts have to be viewed in juxtaposition with the expected change in number of new PhDs. According to one source, the National Institutes of Health found that the number of new PhDs increased by 50% between 2002 and 2009. If this rate were to be sustained, the ratio of jobs to job candidates would decrease even more.

      I plan on researching exit options

      Work-life balance

      According to Yewdell (2009)

      As a graduate student, you should be spending a minimum of 40 hours per week actually designing, performing or interpreting experiments. As there are many other necessary things to do during the day (for example, reading the literature, attending seminars and journal club, talking to colleagues both formally and informally, and common laboratory jobs), this means you will be spending 60 or more hours per week in science-associated activities.

      This is corroborated by career coach Marty Nemko, who wrote

      You spend most of your 60-to-70-hour workweek alone in a lab or at your desk, with little people contact.

      Biomedical researchers who stay in academia are often constrained with respect to the geographic location where they can get jobs. See our writeup on job location options for academics.

      Earnings

      Getting a PhD in a biomedical research field takes 6 to 7 years, during which one makes substantially less money than one could otherwise make. It’s been reported that the average biology PhD had $45k in debt as of 2004.

      Salaries rise afterward, but not rapidly: as of 2009, the starting salary for a postdoc was ~$37k/year (pg. 141), and postdoctoral appointments last 4 years.

      According to the Bureau of Labor Statistics

      Colleges, Universities, and Professional Schools are next in employment, and pay a mean wage of $61,320 per year. Completing the five areas with the most employment are Pharmaceutical and Medicine Manufacturing ($92,130), General Medical and Surgical Hospitals ($80,090) and Drugs and Druggists’ Sundries Merchant Wholesalers ($93,090).

      The “Colleges, Universities, and Professional Schools” category includes postdocs: if one considers professors only, the figure will be more like $80k/year.

      According to Yewdell (2009)

      If you do achieve the ‘Holy Grail’ of full professorship then you will not be poor, but you will be far worse off financially than nearly all of your peers who have similar levels of talent, energy and dedication, but who chose other careers.

      Career coach Marty Nemko wrote

      “According to MIT faculty member Philip Greenspun, Adjusted for IQ, quantitative skills, and working hours, jobs in science are the lowest paid in the United States….”

      A small number of biomedical researchers command high salaries: for example, one source reports that there are 20 in the country with earnings at the $240k+ level.

      Some sources report that biomedical researchers can become very wealthy if as early employees of successful biotech startups, but this is very rare.

      Social Value

      Historically, a large fraction of increase in lifespan and quality of life has been due to biomedical research (e.g. vaccines). Yewdell (2009) wrote

      Society desperately needs your talents […] For rationally thinking people with an altruistic bent, life can be no more rewarding than when practising the scientific method for the benefit of all of the denizens of this fragile planet.

      Some points to keep in mind in assessing the social value of biomedical research are

      • Diminishing returns  Much of the increase in lifespan between 1950 and now was due to cardiovascular disease research, with the gains mostly halting by 1990. There have been significant advances in recent years, such as AIDS treatment drugs, statins, psychiatric drugs. But one should expect the increase in quality of life and lifespan per researcher to go down over time, because of low hanging fruit being plucked, barring radical advances coming from anti-aging research and unexpected sources.
      • Low replication rates — The fact that large fraction of studies don’t replicate suggesting that much research doesn’t move science forward.
      • Power law distribution of research contributions A small fraction of researchers produce 100x+ as much value as the average researcher. To the extent that success is driven by skill rather than luck, prospects for impact depend heavily on your ability.

      80,000 Hours plans to publish an overview of biomedical research that will address the social value of going into biomedical research in more detail.

      See also

      Biomedical Research Workforce Working Group Report (2012) by the National Institutes of Health.

      How to succeed in science: a concise guide for young biomedical scientists. Part I: taking the plunge (2009) by Jonathan Yewdell.